Как по координатам определить коллинеарность векторов: Коллинеарность векторов, условия коллинеарности векторов.

Содержание

Коллинеарность векторов, условия коллинеарности векторов.

Условия коллинеарности векторов

Два вектора будут коллинеарны при выполнении любого из этих условий:

Условие коллинеарности векторов 1. Два вектора a и b коллинеарны, если существует число n такое, что

a = n · b

Условия коллинеарности векторов 2. Два вектора коллинеарны, если отношения их координат равны.

N.B. Условие 2 неприменимо, если один из компонентов вектора равен нулю.

N.B. Условие 3 применимо только для трехмерных (пространственных) задач.

Примеры задач на коллинеарность векторов


Примеры задач на коллинеарность векторов на плоскости

Пример 1. Какие из векторов a = {1; 2}, b = {4; 8}, c = {5; 9} коллинеарны?

Решение: Так как вектора не содержат компоненты равные нулю, то воспользуемся вторым условием коллинеарности, которое в случае плоской задачи для векторов a и b примет вид:

Значит:

Вектора a и b коллинеарны т.
к.  
1  =  2 .
4 8
Вектора a и с не коллинеарны т.к.   1  ≠  2 .
5 9
Вектора с и b не коллинеарны т.к.   5  ≠  9 .
4 8
Пример 2. Доказать что вектора a = {0; 3} и b = {0; 6} коллинеарны.

Решение: Так как вектора содержат компоненты равные нулю, то воспользуемся первым условием коллинеарности, найдем существует ли такое число n при котором:

b = na.

Для этого найдем ненулевой компонент вектора a в данном случае это ay. Если вектора колинеарны то

n =  by  =  6  = 2
ay 3

Найдем значение na:

na = {2 · 0; 2 · 3} = {0; 6}

Так как b = na, то вектора a и b коллинеарны.

Пример 3. найти значение параметра n при котором вектора a = {3; 2} и b = {9; n} коллинеарны.

Решение: Так как вектора не содержат компоненты равные нулю, то воспользуемся вторым условием коллинеарности

Значит:

Решим это уравнение:

Ответ: вектора a и b коллинеарны при n = 6.


Примеры задач на коллинеарность векторов в пространстве

Пример 4. Какие из векторов a = {1; 2; 3}, b = {4; 8; 12}, c = {5; 10; 12} коллинеарны?

Решение: Так как вектора не содержат компоненты равные нулю, то воспользуемся вторым условием коллинеарности, которое в случае пространственной задачи для векторов a и b примет вид:

ax  =  ay  =  az .
bx by bz

Значит:

Вектора a и b коллинеарны т. к. 14 = 28 = 312

Вектора a и с не коллинеарны т.к.  15 = 210 ≠ 312

Вектора с и b не коллинеарны т.к. 54 = 108 ≠ 1212

Пример 5. Доказать что вектора a = {0; 3; 1} и b = {0; 6; 2} коллинеарны.

Решение: Так как вектора содержат компоненты равные нулю, то воспользуемся первым условием коллинеарности, найдем существует ли такое число n при котором:

b = na.

Для этого найдем ненулевой компонент вектора a в данном случае это ay. Если вектора колинеарны то

n =  by  =  6  = 2
ay 3

Найдем значение na:

na = {2 · 0; 2 · 3; 2 · 1} = {0; 6; 2}

Так как b = na, то вектора a и b коллинеарны.

Пример 6. найти значение параметров n и m при которых вектора a = {3; 2; m} и b = {9; n; 12} коллинеарны.

Решение: Так как вектора не содержат компоненты равные нулю, то воспользуемся вторым условием коллинеарности

ax  =  ay  =  az .
bx by bz

Значит:

Из этого соотношения получим два уравнения:

Решим эти уравнения:

Ответ: вектора a и b коллинеарны при n = 6 и m = 4.

Коллинеарность векторов, условия коллинеарности векторов.

Условия коллинеарности векторов

Два вектора будут коллинеарны при выполнении любого из этих условий: Условие коллинеарности векторов 1. Два вектора a и b коллинеарны, если существует число n такое, что

a = n · b

Условия коллинеарности векторов 2. Два вектора коллинеарны, если отношения их координат равны.

N.B. Условие 2 неприменимо, если один из компонентов вектора равен нулю.

N.B. Условие 3 применимо только для трехмерных (пространственных) задач.

Доказательство третего условия коллинеарности

Пусть есть два коллинеарные вектора a = {ax; ay; az} и b = {nax; nay; naz}. Найдем их векторное произведение

a × b = ijk = i (aybz — azby) — j (axbz — azbx) + k (axby — aybx) = 
 ax  ay  az 
 bx  by  bz 

= i (aynaz — aznay) — j (axnaz — aznax) + k (axnay — aynax) = 0i + 0j + 0k = 0

Примеры задач на коллинеарность векторов на плоскости

Пример 1. Какие из векторов a = {1; 2}, b = {4; 8}, c = {5; 9} коллинеарны?

Решение: Так как вектора не содержат компоненты равные нулю, то воспользуемся вторым условием коллинеарности, которое в случае плоской задачи для векторов a и b примет вид:

Значит:

Вектора a и b коллинеарны т. к.  1 = 2.
48
Вектора a и с не коллинеарны т.к.  1 ≠ 2.
59
Вектора с и b не коллинеарны т.к.  5 ≠ 9.
48
Пример 2. Доказать что вектора a = {0; 3} и b = {0; 6} коллинеарны.

Решение: Так как вектора содержат компоненты равные нулю, то воспользуемся первым условием коллинеарности, найдем существует ли такое число n при котором:

b = na.

Для этого найдем ненулевой компонент вектора a в данном случае это ay. Если вектора колинеарны то

n = by = 6 = 2
ay3

Найдем значение na:

na = {2 · 0; 2 · 3} = {0; 6}

Так как b = na, то вектора a и b коллинеарны.

Пример 3. найти значение параметра n при котором вектора a = {3; 2} и b = {9; n} коллинеарны.

Решение: Так как вектора не содержат компоненты равные нулю, то воспользуемся вторым условием коллинеарности

Значит:

Решим это уравнение:

Ответ: вектора a и b коллинеарны при n = 6.

Примеры задач на коллинеарность векторов в пространстве

Пример 4. Какие из векторов a = {1; 2; 3}, b = {4; 8; 12}, c = {5; 10; 12} коллинеарны?

Решение: Так как вектора не содержат компоненты равные нулю, то воспользуемся вторым условием коллинеарности, которое в случае пространственной задачи для векторов a и b примет вид:

ax = ay = az.
bxbybz

Значит:

Вектора a и b коллинеарны т. к.  1 = 2 = 3.
4812
Вектора a и с не коллинеарны т.к.  1 = 2
 ≠ 
3.
51012
Вектора с и b не коллинеарны т.к.  5 = 10 ≠ 12.
4812
Пример 5. Доказать что вектора a = {0; 3; 1} и b = {0; 6; 2} коллинеарны.

Решение: Так как вектора содержат компоненты равные нулю, то воспользуемся первым условием коллинеарности, найдем существует ли такое число n при котором:

b = na.

Для этого найдем ненулевой компонент вектора a в данном случае это ay. Если вектора колинеарны то

n = by = 6 = 2
ay3

Найдем значение na:

na = {2 · 0; 2 · 3; 2 · 1} = {0; 6; 2}

Так как b = na, то вектора a и b коллинеарны.

Пример 6. найти значение параметров n и m при которых вектора a = {3; 2; m} и b = {9; n; 12} коллинеарны.

Решение: Так как вектора не содержат компоненты равные нулю, то воспользуемся вторым условием коллинеарности

ax = ay = az.
bxbybz

Значит:

Из этого соотношения получим два уравнения:

Решим эти уравнения:

Ответ: вектора a и b коллинеарны при n = 6 и m = 4.

Условие коллинеарности векторов, когда векторы параллельны, свойства коллинеарных векторов

В статье ниже рассмотрим условия, при которых векторы считаются коллинеарными, а также разберем тему на конкретных примерах. И, прежде чем приступить к обсуждению, напомним некоторые определения.

Определение 1

Коллинеарные векторы – ненулевые векторы, лежащие на одной прямой или на параллельных прямых. Нулевой вектор считается коллинеарным любому другому.

Данное определение дает возможность убедиться в коллинеарности векторов в их геометрическом отображении, однако точность такого способа может иметь погрешности, например, в зависимости, от качества самого чертежа. Поэтому обратимся к алгебраическому толкованию: сформируем условие, которое будет явным признаком коллинеарности.

Согласно схемам операций над векторами умножение вектора на некоторое заданное число приводит к соответствующему сжатию или растяжению вектора при сохранении или смене направления. Тогда вектор b→=λ·a→ коллинеарен вектору a→ , где λ – некоторое действительное число. Справедливым будет и обратное утверждение: если вектор b→ коллинеарен вектору a→, его можно представить в виде λ·a→. Это является необходимым и достаточным условием коллинеарности двух ненулевых векторов.

Определение 2

Для коллинеарности двух векторов необходимо и достаточно, чтобы они были связаны равенствами: b→=λ·a→ или a→=μ·b→,   μ∈R

Координатная форма условия коллинеарности векторов

Исходные данные: вектор a→ задан в некоторой прямоугольной системе координат на плоскости и имеет координаты (ax, ay), тогда, согласно полученному выше условию, вектор b→=λ·a→ имеет координаты (λ·ax, λ·ay).

По аналогии: если вектор a→ задан в трехмерном пространстве, то он будет представлен в виде координат a=(ax, ay, az) , а вектор b→=λ·a→ имеет координаты (λ·ax, λ·ay, λ·az). Из полученных утверждений следуют условия коллинеарности двух векторов в координатном толковании.

Определение 3
  1.   ​​​Для коллинеарности двух ненулевых векторов на плоскости необходимо и достаточно, чтобы их координаты были связаны соотношениями:  bx=λ·axby=λ·ay  или ax=μ·bxay=μ·by
  2. Для коллинеарности двух ненулевых векторов в пространстве необходимо и достаточно, чтобы их координаты были связаны соотношениями:   bx=λ·axby=λ·ay bz=λ·azили ax=μ·bxay=μ·by az=μ·bz

Мы можем также получить еще одно условие коллинеарности векторов, опираясь на понятие их произведения.

Если ненулевые векторы a→=(ax, ay, az) и b→=(bx, by, bz) коллинеарны, то согласно векторному определению произведения a→×b→=0→. И это также соответствует равенству: i→j→k→axayazbxbybz=0→, что, в свою очередь, возможно только тогда, когда заданные векторы связаны соотношениями b→=λ·a→ и a→=μ·b→ , где μ — произвольное действительное число (на основании теоремы о ранге матрицы), что указывает на факт коллинеарности векторов.

Определение 4

Два ненулевых вектора коллинеарны тогда и только тогда, когда их векторное произведение равно нулевому вектору.

Рассмотрим применение условия коллинеарности на конкретных примерах.

Пример 1

Исходные данные: векторы  a→=(3-22, 1) и b→=(12+1, 2+1) . Необходимо определить, коллинеарны ли они.

Решение

Выполним задачу, опираясь на условие коллинеарности векторов на плоскости в координатах: bx=λ·axby=λ·ay Подставив заданные значения координат, получим: bx=λ·ax⇔12+1=λ·(3-22)⇒λ=1(2+1)·(3-22)=132-4+3-22=12-1by=λ·ay⇔2+1=12-1·1⇔(2+1)·(2-1)=1 ⇔1≡1

Т. е. b→=12-1·a→, следовательно, заданные векторы коллинеарны.

Ответ: заданные векторы коллинеарны.

Нужна помощь преподавателя?

Опиши задание — и наши эксперты тебе помогут!

Описать задание Пример 2

Исходные данные: векторы a→=(1, 0, -2) и b→=(-3, 0, 6) . Необходимо убедиться в их коллинеарности.

Решение

Т.к. bx=λ·axby=λ·ay bz=λ·az⇔-3=-3·10=-3·06=-3·(-2) , то верным будет равенство: b→=-3·a→ , что является необходимым и достаточным условием коллинеарности. Таким образом, заданные векторы коллинеарны.

Найдем также векторное произведение заданных векторов и убедимся, что оно равно нулевому вектору: a→×b→=i→j→k→axayazbxbybz=i→j→k→10-2-306=i→·0·6+j→·(-2)·(-3)+k→·1·0-k→·0·(-3)-j→·1·6-i→·(-2)·0=0→Ответ: заданные векторы коллинеарны.

Пример 3

Исходные данные: векторы a→=(2, 7) и b→=(p, 3) . Необходимо определить, при каком значении pзаданные векторы будут коллинеарны.

Решение

Согласно выведенному выше условию, векторы коллинеарны, если

b→=λ·a→⇔bx=λ·axby=λ·ay⇔p=λ·23=λ·7

тогда λ=37, а p=λ·2⇔p=67 . 

 Ответ: при p=67 заданные векторы коллинеарны.

Также распространены задачи на нахождения вектора, коллинеарного заданному. Решаются они без затруднений, основываясь на условии коллинеарности: : достаточным будет взять произвольное действительное число λ и определить вектор, коллинеарный данному.

Пример 4

Исходные данные: вектор a→=(2, -6) . Необходимо найти любой ненулевой вектор, коллинеарный заданному.

Решение

Ответом может послужить, например, 12·a→=(1, -3) или вектор 3·a→=(6, -18) .

Ответ: вектор, коллинеарный заданному имеет координаты (1, -3).

Пример 5

Исходные данные: вектор a→=(3, 4, -5) . Необходимо определить координаты вектора единичной длины, коллинеарного заданному.

Решение

Вычислим длину заданного вектора по его координатам: a→=ax2+bx2+cx2=32+42+(-5)2=52 Разделим каждую из заданных координат на полученную длину и получим единичный вектор, коллинеарный данному: 1a→·a→=(352, 452,- 12)

Ответ: (352, 452,- 12)

Автор: Ирина Мальцевская

Преподаватель математики и информатики. Кафедра бизнес-информатики Российского университета транспорта

Векторы и операции над векторами

Будут и задачи для самостоятельного решения, к которым можно посмотреть ответы.

Прежде чем Вы узнаете всё о векторах и операциях над ними, настройтесь на решение несложной задачи. Есть вектор Вашей предприимчивости и вектор Ваших инновационных способностей. Вектор предприимчивости ведёт Вас к Цели 1, а вектор инновационных способностей — к Цели 2. Правила игры таковы, что Вы не можете двигаться сразу по направлениям двух этих векторов и достигнуть сразу двух целей. Векторы взаимодействуют, или, если говорить математическим языком, над векторами производится некоторая операция. Результатом этой операции становится вектор «Результат», который приводит Вас к Цели 3.

А теперь скажите: результатом какой операции над векторами «Предприимчивость» и «Инновационные способности» является вектор «Результат»? Если не можете сказать сразу, не унывайте. По мере изучения этого урока Вы сможете ответить на этот вопрос.

Как мы уже увидели выше, вектор обязательно идёт от некоторой точки A по прямой к некоторой точке B. Следовательно, каждый вектор имеет не только числовое значение — длину, но также физическое и геометрическое — направленность. Из этого выводится первое, самое простое определение вектора. Итак, вектор — это направленный отрезок, идущий от точки A к точке B. Обозначается он так: .

А чтобы приступить к различным операциям с векторами, нам нужно познакомиться с ещё одним определением вектора.

Вектор — это вид представления точки, до которой требуется добраться из некоторой начальной точки. Например, трёхмерный вектор, как правило, записывается в виде (х, y, z). Говоря совсем просто, эти числа означают, как далеко требуется пройти в трёх различных направлениях, чтобы добраться до точки.

Пусть дан вектор. При этом x = 3 (правая рука указывает направо), y = 1 (левая рука указывает вперёд), z = 5 (под точкой стоит лестница, ведущая вверх). По этим данным вы найдёте точку, проходя 3 метра в направлении, указываемом правой рукой, затем 1 метр в направлении, указываемом левой рукой, а далее Вас ждёт лестница и, поднимаясь на 5 метров, Вы, наконец, окажетесь в конечной точке.

Все остальные термины — это уточнения представленного выше объяснения, необходимые для различных операций над векторами, то есть, решения практических задач. Пройдёмся по этим более строгим определениям, останавливаясь на типичных задачах на векторы.


Физическими примерами векторных величин могут служить смещение материальной точки, двигающейся в пространстве, скорость и ускорение этой точки, а также действующая на неё сила.


Геометрический вектор представлен в двумерном и трёхмерном пространстве в виде направленного отрезка. Это отрезок, у которого различают начало и конец.

Если A — начало вектора, а B — его конец, то вектор обозначается символом или одной строчной буквой . На рисунке конец вектора указывается стрелкой (рис. 1)


Длиной (или модулем) геометрического вектора называется длина порождающего его отрезка


Два вектора называются равными, если они могут быть совмещены (при совпадении направлений) путём параллельного переноса, т.е. если они параллельны, направлены в одну и ту же сторону и имеют равные длины.


В физике часто рассматриваются закреплённые векторы, заданные точкой приложения, длиной и направлением. Если точка приложения вектора не имеет значения, то его можно переносить, сохраняя длину и направление в любую точку пространства. В этом случае вектор называется свободным. Мы договоримся рассматривать только свободные векторы.

Умножение вектора на число


Сложение и вычитание векторов

Слагаемые называются составляющими вектора , а сформулированное правило — правилом многоугольника. Этот многоугольник может и не быть плоским.

Пример 1. Упростить выражение:

.

Решение:

,

то есть, векторы можно складывать и умножать на числа так же, как и многочлены (в частности, также задачи на упрощение выражений). Обычно необходимость упрощать линейно подобные выражения с векторами возникает перед вычислением произведений векторов.

Пример 2. Векторы и служат диагоналями параллелограмма ABCD (рис. 4а). Выразить через и векторы , , и , являющиеся сторонами этого параллелограмма.

Решение. Точка пересечения диагоналей параллелограмма делит каждую диагональ пополам. Длины требуемых в условии задачи векторов находим либо как половины сумм векторов, образующих с искомыми треугольник, либо как половины разностей (в зависимости от направления вектора, служащего диагональю), либо, как в последнем случае, половины суммы, взятой со знаком минус. Результат — требуемые в условии задачи векторы:

Есть все основания полагать, что теперь Вы правильно ответили на вопрос о векторах «Предприимчивость» и «Инновационные способности» в начале этого урока. Правильный ответ: над этими векторами производится операция сложения.

Решить задачи на векторы самостоятельно, а затем посмотреть решения

Как найти длину суммы векторов?

Эта задача занимает особое место в операциях с векторами, так как предполагает использование тригонометрических свойств. Допустим, Вам попалась задача вроде следующей:

Даны длины векторов и длина суммы этих векторов . Найти длину разности этих векторов .

Решения этой и других подобных задач и объяснения, как их решать — в уроке «Сложение векторов: длина суммы векторов и теорема косинусов«.

А проверить решение таких задач можно на Калькуляторе онлайн «Неизвестная сторона треугольника (сложение векторов и теорема косинусов)».

А где произведения векторов?

Произведения вектора на вектор не являются линейными операциями и рассматриваются отдельно. И у нас есть уроки «Скалярное произведение векторов» и «Векторное и смешанное произведения векторов».

Проекция вектора на ось равна произведению длины проектируемого вектора на косинус угла между вектором и осью:

Как известно, проекцией точки A на прямую (плоскость) служит основание перпендикуляра , опущенного из этой точки на прямую (плоскость).

Пусть — произвольный вектор (Рис. 5), а и — проекции его начала (точки A) и конца (точки B) на ось l. (Для построения проекции точки A) на прямую проводим через точку A плоскость, перпендикулярную прямой. Пересечение прямой и плоскости определит требуемую проекцию.

Составляющей вектора на оси l называется такой вектор , лежащий на этой оси, начало которого совпадает с проекцией начала, а конец — с проекцией конца вектора .

Проекцией вектора на ось l называется число

,

равное длине составляющего вектора на этой оси, взятое со знаком плюс, если направление составляюшей совпадает с направлением оси l, и со знаком минус, если эти направления противоположны.

Основные свойства проекций вектора на ось:

1. Проекции равных векторов на одну и ту же ось равны между собой.

2. При умножении вектора на число его проекция умножается на это же число.

3. Проекция суммы векторов на какую-либо ось равна сумме проекций на эту же ось слагаемых векторов.

4. Проекция вектора на ось равна произведению длины проектируемого вектора на косинус угла между вектором и осью:

Пример 5. Рассчитать проекцию суммы векторов на ось l, если , а углы —

.

Решение. Спроектируем векторы на ось l как определено в теоретической справке выше. Из рис.5а очевидно, что проекция суммы векторов равна сумме проекций векторов. Вычисляем эти проекции:

Находим окончательную проекцию суммы векторов:

.

Знакомство с прямоугольной декартовой системой координат в пространстве состоялось в соответствующем уроке, желательно открыть его в новом окне.

В упорядоченной системе координатных осей 0xyz ось Ox называется осью абсцисс, ось 0yосью ординат, и ось 0zосью аппликат.

С произвольной точкой М  пространства свяжем вектор

,

называемый радиус-вектором точки М и спроецируем его на каждую из координатных осей. Обозначим величины соответствующих проекций:

Числа x, y, z называются координатами точки М , соответственно абсциссой, ординатой и аппликатой, и записываются в виде упорядоченной точки чисел: M (x; y; z) (рис.6).

Вектор единичной длины, направление которого совпадает с направлением оси, называют единичным вектором(или ортом) оси. Обозначим через



Соответственно орты координатных осей Ox, Oy, Oz


Теорема. Всякий вектор может быть разложен по ортам координатных осей:

        (2)

Равенство (2) называется разложением вектора по координатным осям. Коэффициентами этого разложения являются проекции вектора на координатные оси. Таким образом, коэффициентами разложения (2) вектора по координатным осям являются координаты вектора.

После выбора в пространстве определённой системы координат вектор и тройка его координат однозначно определяют друг друга, поэтому вектор может быть записан в форме

              (3)

Представления вектора в виде (2) и (3) тождественны.

Как мы уже отмечали, векторы называются коллинеарными, если они связаны отношением

.

Пусть даны векторы . Эти векторы коллинеарны, если координаты векторов связаны отношением

,

то есть, координаты векторов пропорциональны.

Пример 6. Даны векторы . Коллинеарны ли эти векторы?

Решение. Выясним соотношение координат данных векторов:

.

Координаты векторов пропорциональны, следовательно, векторы коллинеарны, или, что то же самое, параллельны.

Вследствие взаимной перпендикулярности координатных осей длина вектора

равна длине диагонали прямоугольного параллелепипеда, построенного на векторах

и выражается равенством

                       (4)

Вектор полностью определяется заданием двух точек (начала и конца), поэтому координаты вектора можно выразить через координаты этих точек.

Пусть в заданной системе координат начало вектора находится в точке

а конец – в точке

(рис. 8).

Тогда

Из равенства


следует, что

Отсюда

или в координатной форме

          (5)

Следовательно, координаты вектора равны разностям одноимённых координат конца и начала вектора. Формула (4) в этом случае примет вид

          (6)

Направление вектора определяют направляющие косинусы. Это косинусы углов, которые вектор образует с осями Ox, Oy и Oz. Обозначим эти углы соответственно α, β и γ. Тогда косинусы этих углов можно найти по формулам

,

,

.

Направляющие косинусы вектора являются также координатами орта этого вектора и, таким образом, орт вектора

или

.

Учитывая, что длина орта вектора равна одной единице, то есть

,

получаем следующее равенство для направляющих косинусов:

.

Пример 7. Найти длину вектора x = (3; 0; 4).

Решение. Длина вектора равна

Пример 8. Даны точки:

Выяснить, равнобедренный ли треугольник, построенный на этих точках.

Решение. По формуле длины вектора (6) найдём длины сторон и установим, есть ли среди них две равные:

Две равные стороны нашлись, следовательно необходимость искать длину третьей стороны отпадает, а заданный треугольник является равнобедренным.

Пример 9. Найти длину вектора и его направляющие косинусы, если .

Решение. Координаты вектора даны:

.

Длина вектора равна квадратному корню из суммы квадратов координат вектора:

.

Находим направляющие косинусы:

Решить задачу на векторы самостоятельно, а затем посмотреть решение

Пусть даны два вектора и , заданные своими проекциями:

или

или 

Укажем действия над этими векторами.

1.Сложение:

или, что то же

(при сложении двух векторов одноимённые координаты складываются).

2.Вычитание:

или, что то же

,

(при вычитании двух векторов одноимённые координаты вычитаются).

3.Умножение вектора на число:

или, что то же

,

(при умножении вектора на число все координаты умножаются на это число).

Пример 11. Даны два вектора, заданные координатами:

.

Найти заданный координатами вектор, являющийся суммой этих векторов: .

Решение:

.

Решить задачи на координаты векторов самостоятельно, а затем посмотреть решение

При изучении многих вопросов, в частности, экономических, оказалось удобным обобщить рассмотренные приёмы установления соответствия между числами и точками двумерного и трёхмерного пространства и рассматривать последовательности n действительных чисел как «точки» некоторого абстрактного «n-мерного пространства», а сами числа — как «координаты» этих точек. За составляющие n-мерного вектора можно принимать такие данные, как урожайность различных культур, объёмы продаж товаров, технические коэффициенты, номенклатура товаров на складах и т.д.

n-мерным вектором называется упорядоченный набор из n действительных чисел, записываемых в виде

,

где  - i – й элемент (или i – я координата) вектора x.

Возможна и другая запись вектора – в виде столбца координат:

Размерность вектора определяется числом его координат и является его отличительной характеристикой. Например, (2; 5) – двухмерный вектор, (2; -3; 0) – трёхмерный, (1; 3; -2; -4; 7) – пятимерный,

n – мерный вектор.

Нулевым вектором называется вектор, все координаты которого равны нулю:

0 = (0; 0; …; 0).

Введём операции над n-мерными векторами.

Произведением вектора


на действительное число  называется вектор

(при умножении вектора на число каждая его координата умножается на это число).

Зная вектор

можно получить противоположный вектор

Суммой векторов

и

называется вектор

,

(при сложении векторов одной и той же размерности их соответствующие координаты почленно складываются).

Если в плане продаж сети торговых предприятий продажи товаров определить как положительные уровни товаров, а затраты на продажи – как отрицательные, то получим вектор затрат-продаж

,

где

продажи (затраты) k – м предприятием товара i, а k = 1, 2, 3,…, m .

Суммарный вектор затрат-продаж y определяется суммированием векторов затрат-продаж всех m предприятий сети:

Сумма противоположных векторов даёт нулевой вектор:

При вычитании двух векторов одной и той же размерности их соответствующие координаты почленно вычитаются:

Операции над n-мерными векторами удовлетворяют следующим свойствам.

Свойство 1.


Свойство 2.

Свойство 3.

Свойство 4.

Свойство 5.

Свойство 6.

Поделиться с друзьями

Весь блок «Аналитическая геометрия»

  • Векторы
  • Плоскость
  • Прямая на плоскости

Векторное Произведение Векторов.

Свойства, определение

Определение векторного произведения

Система координат — способ определить положение и перемещение точки или тела с помощью чисел или других символов.

Координаты — это совокупность чисел, которые определяют положение какого-либо объекта на прямой, плоскости, поверхности или в пространстве. Как найти координаты точки мы рассказали в этой статье.

Скаляр — это величина, которая полностью определяется в любой координатной системе одним числом или функцией.

Вектор — направленный отрезок прямой, для которого указано, какая точка является началом, а какая — концом.


Вектор с началом в точке A и концом в точке B принято обозначать как →AB. Векторы также можно обозначать малыми латинскими буквами со стрелкой или черточкой над ними, вот так: →a.

Коллинеарность — отношение параллельности векторов. Два ненулевых вектора называются коллинеарными, если они лежат на параллельных прямых или на одной прямой.

Проще говоря это «параллельные» векторы. Коллинеарные векторы могут быть одинаково направлены или противоположно направлены. Основное обозначение — →a || →b. Сонаправленные коллинеарные векторы обозначаются так →a ↑↑ →b, противоположно направленные — →a ↑↓ →b.

Прежде чем дать определение векторного произведения, разберемся с ориентацией упорядоченной тройки векторов →a, →b, →c в трехмерном пространстве.

Отложим векторы →a, →b, →c от одной точки. В зависимости от направления вектора →c тройка →a, →b, →c может быть правой или левой.

Посмотрим с конца вектора →c на то, как происходит кратчайший поворот от вектора →a к →b. Если кратчайший поворот происходит против часовой стрелки, то тройка векторов →a, →b, →c называется правой, по часовой стрелке — левой.



Теперь возьмем два неколлинеарных вектора →a и →b. Отложим от точки А векторы →AB = →a и →AC = →b. Построим некоторый вектор →AD = →c, перпендикулярный одновременно и →AB и →AC.

Очевидно, что при построении вектора →AD = →c мы можем поступить по-разному, если зададим ему либо одно направление, либо противоположное.


В зависимости от направления вектора →AD = →c упорядоченная тройка векторов →a, →b, →c может быть правой или левой.

И сейчас мы подошли к определению векторного произведения. Оно дается для двух векторов, которые заданы в прямоугольной системе координат трехмерного пространства.

Векторным произведением двух векторов →a и →b, которые заданы в прямоугольной системе координат трехмерного пространства, называется такой вектор →c, что:

  • он является нулевым, если векторы →a и →b коллинеарны;
  • он перпендикулярен и вектору →a и вектору →b;
  • длина векторного произведения равна произведению длин векторов →a и →b на синус угла между ними
  • тройка векторов →a, →b, →c ориентирована так же, как и заданная система координат.

Векторным произведением вектора →a на вектор →b называется вектор →c, длина которого численно равна площади параллелограмма построенного на векторах →a и →b, перпендикулярный к плоскости этих векторов и направленный так, чтобы наименьшее вращение от →a к →b вокруг вектора c осуществлялось против часовой стрелки, если смотреть с конца вектора →c.


Векторное произведение двух векторов a = {ax; ay; az} и b = {bx; by; bz} в декартовой системе координат — это вектор, значение которого можно вычислить, используя формулы вычисления векторного произведения векторов:



Векторное произведение векторов →a и →b обозначается как [→a • →b].

Другое определение связано с правой рукой человека, откуда и есть название. На рисунке тройка векторов →a, →b, [→a • →b] является правой.


Еще есть аналитический способ определения правой и левой тройки векторов — он требует задания в рассматриваемом пространстве правой или левой системы координат, причём не обязательно прямоугольной и ортонормированной.

Нужно составить матрицу, первой строкой которой будут координаты вектора →a, второй — вектора →b, третьей — вектора →c. Затем, в зависимости от знака определителя этой матрицы, можно сделать следующие выводы:

  • Если определитель положителен, то тройка векторов имеет ту же ориентацию, что и система координат.
  • Если определитель отрицателен, то тройка векторов имеет ориентацию, противоположную ориентации системы координат.
  • Если определитель равен нулю, то векторы компланарны (линейно зависимы).

Координаты векторного произведения

Рассмотрим векторное произведение векторов в координатах.

Сформулируем второе определение векторного произведения, которое позволяет находить его координаты по координатам заданных векторов.

В прямоугольной системе координат трехмерного пространства векторное произведение двух векторов →a = (ax, ay, az) и →b = (bx, by, bz) есть вектор


, где

→i, →j, →k — координатные векторы.

Это определение показывает нам векторное произведение в координатной форме.

Векторное произведение удобно представлять в виде определителя квадратной матрицы третьего порядка, первая строка которой есть орты →i, →j, →k, во второй строке находятся координаты вектора →a, а в третьей — координаты вектора →b в заданной прямоугольной системе координат:


Если разложим этот определитель по элементам первой строки, то получим равенство из определения векторного произведения в координатах:


Важно отметить, что координатная форма векторного произведения согласуется с определением,которое мы дали в первом пункте этой статьи. Более того, эти два определения векторного произведения эквивалентны.

Свойства векторного произведения

Векторное произведение в координатах представляется в виде определителя матрицы:


На основании свойств определителя можно легко обосновать свойства векторного произведения векторов:


  1. Антикоммутативность
  2. Свойство дистрибутивности

    или


  3. Сочетательное свойство

    или


    , где λ произвольное действительное число.

Для большей ясности докажем свойство антикоммутативности векторного произведения.

По определению


и


Нам известно, что значение определителя матрицы изменяется на противоположное, если переставить местами две строки, поэтому


что доказывает свойство антикоммутативности векторного произведения.

Чтобы найти модуль векторного произведения векторов u и v нужно найти площадь параллелограмма, который построен на данных векторах: S = | u × v | = | u | * | v | * sinθ, где θ — угол между векторами.

Векторное произведение векторов u и v равно нулевому вектору, если u и v параллельны (коллинеарны): u × v = 0, если u ∥ v (θ = 0).

 

Примеры решения задач

Пример 1

а) Найти длину векторного произведения векторов →a и →b, если |→a| = 2, |→b| = 3, ∠(→a, →b) = π/3.

б) Найти площадь параллелограмма, построенного на векторах →a и →b, если |→a| = 2, |→b| = 3, ∠(→a, →b) = π/3.

Как решаем:

а) По условию требуется найти длину векторного произведения. Подставляем данные в формулу:


Ответ:


Так как в задаче речь идет о длине, то в ответе указываем размерность — единицы.

б) По условию требуется найти площадь параллелограмма, который построен на векторах →a и →b. Площадь такого параллелограмма численно равна длине векторного произведения:


Ответ:


Пример 2

Найти |[-3→a x 2→b]|, если |→a| = 1/2, |→b| = 1/6, ∠(→a, →b) = π/2.

Как решаем:

По условию снова нужно найти длину векторного произведения. Используем нашу формулу:


Согласно ассоциативным законам, выносим константы за переделы векторного произведения.

Выносим константу за пределы модуля, при этом модуль позволяет убрать знак минус. Длина же не может быть отрицательной.

Ответ:


Пример 3

Даны вершины треугольника A (0, 2, 0), B (-2, 5,0), C (-2, 2, 6). Найти его площадь.

Как решаем:

Сначала найдём векторы:


Затем векторное произведение:


Вычислим его длину:


Подставим данные в формулы площадей параллелограмма и треугольника:


Ответ:


Геометрический смысл векторного произведения

По определению длина векторного произведения векторов равна


А из курса геометрии средней школы мы знаем, что площадь треугольника равна половине произведения длин двух сторон треугольника на синус угла между ними.

Поэтому длина векторного произведения равна удвоенной площади треугольника, имеющего сторонами векторы →a и →b, если их отложить от одной точки. Проще говоря, длина векторного произведения векторов →a и →b равна площади параллелограмма со сторонами |→a| и |→b| и углом между ними, равным (→a, →b). В этом состоит геометрический смысл векторного произведения.


Физический смысл векторного произведения

В механике — одном из разделов физики — благодаря векторному произведению можно определить момент силы относительно точки пространства. Поэтому сформулируем еще одно важное определение.

Под моментом силы →F, приложенной к точке B, относительно точки A понимается следующее векторное произведение [→A B × →F].


Вектор линейной скорости →V точки M колеса равен векторному произведению вектора угловой скорости →W и радиус-вектора точки колеса, то есть →V = →W`→rM.


Как доказать параллельность векторов

Вектора, параллельные одной прямой или лежащие на одной прямой называют коллинеарными векторами (рис. 1).

рис. 1

Условия коллинеарности векторов

Два вектора будут коллинеарны при выполнении любого из этих условий:

Условие коллинеарности векторов 1. Два вектора a и b коллинеарны, если существует число n такое, что

N.B. Условие 2 неприменимо, если один из компонентов вектора равен нулю.

N.B. Условие 3 применимо только для трехмерных (пространственных) задач.

Доказательство третего условия коллинеарности

Пусть есть два коллинеарные вектора a = < ax ; ay ; az > и b = < nax ; nay ; naz >. Найдем их векторное произведение

Примеры задач на коллинеарность векторов

Примеры задач на коллинеарность векторов на плоскости

Решение: Так как вектора не содержат компоненты равные нулю, то воспользуемся вторым условием коллинеарности, которое в случае плоской задачи для векторов a и b примет вид:

Вектора a и b коллинеарны т.к.1=2.
48
Вектора a и с не коллинеарны т. к.12.
59
Вектора с и b не коллинеарны т.к.59.
48

Решение: Так как вектора содержат компоненты равные нулю, то воспользуемся первым условием коллинеарности, найдем существует ли такое число n при котором:

Для этого найдем ненулевой компонент вектора a в данном случае это ay . Если вектора колинеарны то

Найдем значение n a :

Так как b = n a , то вектора a и b коллинеарны.

Решение: Так как вектора не содержат компоненты равные нулю, то воспользуемся вторым условием коллинеарности

Решим это уравнение:

Ответ: вектора a и b коллинеарны при n = 6.

Примеры задач на коллинеарность векторов в пространстве

Решение: Так как вектора не содержат компоненты равные нулю, то воспользуемся вторым условием коллинеарности, которое в случае пространственной задачи для векторов a и b примет вид:

Вектора a и b коллинеарны т. к. 1 4 = 2 8 = 3 12

Вектора a и с не коллинеарны т.к. 1 5 = 2 10 ≠ 3 12

Вектора с и b не коллинеарны т.к. 5 4 = 10 8 ≠ 12 12

Решение: Так как вектора содержат компоненты равные нулю, то воспользуемся первым условием коллинеарности, найдем существует ли такое число n при котором:

Для этого найдем ненулевой компонент вектора a в данном случае это ay . Если вектора колинеарны то

Найдем значение n a :

Так как b = n a , то вектора a и b коллинеарны.

Решение: Так как вектора не содержат компоненты равные нулю, то воспользуемся вторым условием коллинеарности

Из этого соотношения получим два уравнения:

Решим эти уравнения:

Ответ: вектора a и b коллинеарны при n = 6 и m = 4.

В статье ниже рассмотрим условия, при которых векторы считаются коллинеарными, а также разберем тему на конкретных примерах. И, прежде чем приступить к обсуждению, напомним некоторые определения.

Коллинеарные векторы – ненулевые векторы, лежащие на одной прямой или на параллельных прямых. Нулевой вектор считается коллинеарным любому другому.

Данное определение дает возможность убедиться в коллинеарности векторов в их геометрическом отображении, однако точность такого способа может иметь погрешности, например, в зависимости, от качества самого чертежа. Поэтому обратимся к алгебраическому толкованию: сформируем условие, которое будет явным признаком коллинеарности.

Согласно схемам операций над векторами умножение вектора на некоторое заданное число приводит к соответствующему сжатию или растяжению вектора при сохранении или смене направления. Тогда вектор b → = λ · a → коллинеарен вектору a → , где λ – некоторое действительное число. Справедливым будет и обратное утверждение: если вектор b → коллинеарен вектору a → , его можно представить в виде λ · a → . Это является необходимым и достаточным условием коллинеарности двух ненулевых векторов.

Для коллинеарности двух векторов необходимо и достаточно, чтобы они были связаны равенствами: b → = λ · a → или a → = μ · b → , μ ∈ R

Координатная форма условия коллинеарности векторов

Исходные данные: вектор a → задан в некоторой прямоугольной системе координат на плоскости и имеет координаты ( a x , a y ) , тогда, согласно полученному выше условию, вектор b → = λ · a → имеет координаты ( λ · a x , λ · a y ) .

По аналогии: если вектор a → задан в трехмерном пространстве, то он будет представлен в виде координат a = ( a x , a y , a z ) , а вектор b → = λ · a → имеет координаты ( λ · a x , λ · a y , λ · a z ) . Из полученных утверждений следуют условия коллинеарности двух векторов в координатном толковании.

  1. ​​​Для коллинеарности двух ненулевых векторов на плоскости необходимо и достаточно, чтобы их координаты были связаны соотношениями: b x = λ · a x b y = λ · a y или a x = μ · b x a y = μ · b y
  2. Для коллинеарности двух ненулевых векторов в пространстве необходимо и достаточно, чтобы их координаты были связаны соотношениями: b x = λ · a x b y = λ · a y b z = λ · a z или a x = μ · b x a y = μ · b y a z = μ · b z

Мы можем также получить еще одно условие коллинеарности векторов, опираясь на понятие их произведения.

Если ненулевые векторы a → = ( a x , a y , a z ) и b → = ( b x , b y , b z ) коллинеарны, то согласно векторному определению произведения a → × b → = 0 → . И это также соответствует равенству: i → j → k → a x a y a z b x b y b z = 0 → , что, в свою очередь, возможно только тогда, когда заданные векторы связаны соотношениями b → = λ · a → и a → = μ · b → , где μ – произвольное действительное число (на основании теоремы о ранге матрицы), что указывает на факт коллинеарности векторов.

Два ненулевых вектора коллинеарны тогда и только тогда, когда их векторное произведение равно нулевому вектору.

Рассмотрим применение условия коллинеарности на конкретных примерах.

Исходные данные: векторы a → = ( 3 – 2 2 , 1 ) и b → = ( 1 2 + 1 , 2 + 1 ) . Необходимо определить, коллинеарны ли они.

Решение

Выполним задачу, опираясь на условие коллинеарности векторов на плоскости в координатах: b x = λ · a x b y = λ · a y Подставив заданные значения координат, получим: b x = λ · a x ⇔ 1 2 + 1 = λ · ( 3 – 2 2 ) ⇒ λ = 1 ( 2 + 1 ) · ( 3 – 2 2 ) = 1 3 2 – 4 + 3 – 2 2 = 1 2 – 1 b y = λ · a y ⇔ 2 + 1 = 1 2 – 1 · 1 ⇔ ( 2 + 1 ) · ( 2 – 1 ) = 1 ⇔ 1 ≡ 1

Т. е. b → = 1 2 – 1 · a → , следовательно, заданные векторы коллинеарны.

Ответ: заданные векторы коллинеарны.

Исходные данные: векторы a → = ( 1 , 0 , – 2 ) и b → = ( – 3 , 0 , 6 ) . Необходимо убедиться в их коллинеарности.

Решение

Т.к. b x = λ · a x b y = λ · a y b z = λ · a z ⇔ – 3 = – 3 · 1 0 = – 3 · 0 6 = – 3 · ( – 2 ) , то верным будет равенство: b → = – 3 · a → , что является необходимым и достаточным условием коллинеарности. Таким образом, заданные векторы коллинеарны.

Найдем также векторное произведение заданных векторов и убедимся, что оно равно нулевому вектору: a → × b → = i → j → k → a x a y a z b x b y b z = i → j → k → 1 0 – 2 – 3 0 6 = i → · 0 · 6 + j → · ( – 2 ) · ( – 3 ) + k → · 1 · 0 – k → · 0 · ( – 3 ) – j → · 1 · 6 – i → · ( – 2 ) · 0 = 0 → Ответ: заданные векторы коллинеарны.

Исходные данные: векторы a → = ( 2 , 7 ) и b → = ( p , 3 ) . Необходимо определить, при каком значении p заданные векторы будут коллинеарны.

Решение

Согласно выведенному выше условию, векторы коллинеарны, если

b → = λ · a → ⇔ b x = λ · a x b y = λ · a y ⇔ p = λ · 2 3 = λ · 7

тогда λ = 3 7 , а p = λ · 2 ⇔ p = 6 7 .

Ответ: при p = 6 7 заданные векторы коллинеарны.

Также распространены задачи на нахождения вектора, коллинеарного заданному. Решаются они без затруднений, основываясь на условии коллинеарности: : достаточным будет взять произвольное действительное число λ и определить вектор, коллинеарный данному.

Исходные данные: вектор a → = ( 2 , – 6 ) . Необходимо найти любой ненулевой вектор, коллинеарный заданному.

Решение

Ответом может послужить, например, 1 2 · a → = ( 1 , – 3 ) или вектор 3 · a → = ( 6 , – 18 ) .

Ответ: вектор, коллинеарный заданному имеет координаты ( 1 , – 3 ) .

Исходные данные: вектор a → = ( 3 , 4 , – 5 ) . Необходимо определить координаты вектора единичной длины, коллинеарного заданному.

Решение

Вычислим длину заданного вектора по его координатам: a → = a x 2 + b x 2 + c x 2 = 3 2 + 4 2 + ( – 5 ) 2 = 5 2 Разделим каждую из заданных координат на полученную длину и получим единичный вектор, коллинеарный данному: 1 a → · a → = ( 3 5 2 , 4 5 2 , – 1 2 )

Общие сведения

Вектором называют направленный отрезок, который имеет начало и конец. Обозначают его либо большими буквами, либо маленькими, например, АБ или a. Над буквой ставится знак вектора — стрелка. Любой отрезок характеризуется длиной, которую называют модулем. Если начало и конец прямой совпадают, то такой вектор носит название нулевой и обозначается в виде точки. При этом его модуль будет равняться нулю.

Для равенства векторов необходимо выполнение двух условий:

  • модули отрезков должны быть равны;
  • сравниваемые отрезки должны быть направлены в одну сторону.

Равные вектора могут быть совмещены параллельным переносом, при этом начало и конец отрезков должны совпадать. Если ограниченные линии не являются равными, но лежат на параллельных прямых, то их называют коллинеарными, то есть, по определению коллинеарных векторов, их направление для определения признака не является важным.

Коллинеарность является одним из признаков сонаправленности, но для выполнения последнего они должны ещё и совпадать по направлению. Наглядным понятием, объясняющим сонаправленность, является прямое движение транспорта или пешехода. Например, если рассматривать две траектории движения как векторы АБ и СД, лежащие на плоскости, при этом их лучи лежат в одной полуплоскости и перпендикулярны её границам, то их можно назвать сонаправленными.

Поэтому параллельные отрезки будут направлены в одну сторону лишь тогда, когда их лучи находятся по одну сторону от прямой, соединяющей их начала. При этом если векторы коллинеарны, но не сонаправлены, то они будут являться противоположными.

С векторами можно выполнять любые простейшие арифметические операции. При сложении используют правила параллелограмма и треугольника. Пусть есть два отрезка, имеющие общее начало. Для того чтобы найти их сумму, необходимо фигуру достроить до параллелограмма. Диагональ этой фигуры и будет искомой величиной. Когда же конец одного отрезка является началом другого, то, соединив свободные точки, можно получить треугольник. Новая прямая и будет являться вектором суммы. Следует отметить, что эти правила равнозначны друг другу. Вычитание отрезков находится аналогично.

Вектор можно и умножить на число, то есть длина отрезка увеличивается на значение множителя. Если в произведении стоит отрицательное число, то характеристика меняет направление.

Критерии коллинеарности

Теорема критерия коллинеарности представляет собой утверждение, которое сообщает, что если есть два не ортогональных отрезка, одинаковых по длине, a и b, то вектор a может быть выражен через формулу a || b = a = y * b. При этом y обозначает любое произвольное число. Есть и обратное утверждение: если вектор b умножить на число и получится отрезок a, то тогда a и b будут коллинеарными.

Эти два правила тождественны и называются критериями коллинеарности. Для их доказательства нужно знать правило арифметических действий с параллельными и перпендикулярными векторами, а также понимать основной базис. Заключается он в том, что если имеются три отрезка a, b и c, при этом верной является следующая комбинация a || b и a || c, то справедливо утверждать, что b || c.

Для того чтобы доказать свойство a || b = a = y * b, нужно воспользоваться определением коллинеарности. Из него следует, что если a || b, то отрезки могут быть сонаправлены или противоположно направлены. Таким образом, необходимо проверить утверждение для двух случаев:

Если предположения окажутся верными, то можно будет сделать вывод о справедливости записи для других случаев. То есть к любым параллельным отрезкам можно применить равенство a = u * b. Этот критерий занимает важное место в геометрии наряду со свойствами перпендикулярности (ортогональности) прямых.

Сонаправленные вектора

Пусть a и b однонаправленные. Введём число y, равное отношению a на b. Так как длина вектора может быть только положительной, то и y = a /b > 0. Состояние вектора, когда он нулевой, является частным случаем и его можно не рассматривать, так как при этом получится равенство 0 = 0. Если длину b умножить на число, то получится новый вектор. Пусть это будет отрезок c, то есть с = y * b. Учитывая свойство коллинеарности, можно утверждать, что между c и b останется параллельность.

По условию известно, что a || b. Исходя из транзитивности отрезков, можно заключить, что и c || b. Теперь необходимо установить их направление. Изначально a и b направлены в одну сторону. Ведённый множитель больше нуля. Это значит, что после умножения направление вектора не изменится, то есть c будет иметь то же направление что и b. Тогда получается, что a || b и c || b. Отсюда следует, что a || с.

Длина вектора c равняется |c| = |u| * |b|. Вместо u можно подставить a / b. В итоге получится |a| * |b| / |b| = |a|. Таким образом, два условия выполняются, и можно утверждать, что с = a. Получается, что для двух любых однонаправленных векторов будет выполняться правило a = u * b.

Противоположные отрезки

Пусть имеется два отрезка a и b, при этом их направления противоположны друг другу. Можно ввести переменную u, которая будет меньше нуля. Тогда справедливо записать u = – |a| / |b| 0, так как |m| ↑↑ |n|. Отсюда u = 240 / 12 = 20.

  • Требуется доказать, что если отрезки a и b не коллинеарны, то a + b и a — b не коллинеарны тоже. Такие задачи решаются методом от обратного. Для повышения комфортности решения рекомендуется выполнить векторный рисунок в линейных координатах. Делают предположение, что a + b и a — b коллинеарны. Тогда должно выполняться следующее равенство: a + b = u (a — b). При этом u не должно равняться нулю. В выражении можно раскрыть скобки: a + b = u * a — u * b, а затем перенести в левую часть равенства одночлены, содержащие вектор a, а в правую часть — b: a — u * a = – u * b — b. Используя законы умножения, выражение можно преобразовать до вида: (1-u) * a = (-u — 1) * b. После ряда стандартных упрощений получится: a = (-u — 1) * b / 1- u, a b = (u + 1) * b / 1- u. Изучив полученное выражение, можно отметить, что u = 1 противоречит условию, так как a + b = a — b, то b = -b = 0.
  • Установить, являются ли отрезки с1 и с2 коллинеарными по векторам a и b при условии a = <1; 4; -2>, b = <1; 1; -1>; c 1 = a + b, c 2 = 4 a +2 b. Решение выполняют следующим образом. Если векторы коллинеарны, то будет существовать такое число, при котором будет верным равенство: c 1 = u * c 2. Иными словами, векторы коллинеарны если их координаты пропорциональны. Используя исходные данные, получим: c 1 = a + b = <1+1; 4+1; -2+(-1)>= <2; 5; -3>; c 2 = 4 * a + 2 * b = <4*1 + 2*1; 4*4 + 2*1; 4 * (-2) + 2 * (-1)>= <6; 18; -10>. В результате: 2/6 ≠ 5/18 ≠ -3/-10. Отсюда можно сделать вывод, что рассматриваемые отрезки не коллинеарны.
  • Использование онлайн-калькулятора

    Решение простых заданий из школьного курса обычно не вызывает сложностей. Но на практике приходится сталкиваться со сложными выражениями. Для их вычисления нужно проявить усидчивость и при этом быть предельно внимательным. Кроме этого, расчёт занимает довольно много времени, а любая, казалось бы, незначительная оплошность, приведёт к неправильному решению.

    Поэтому условие коллинеарности векторов удобно проверять на так называемых онлайн-калькуляторах. Это обычно мощные сервисы, основная деятельность которых заключается в предоставлении услуг по автоматизации вычислений. Среди них попадаются и сайты, умеющие вычислять и вектора.

    Для того чтобы выполнить на них математические операции, необходимо иметь доступ к интернету и установленный веб-обозреватель. Всё, что требуется от пользователя, это просто зайти на сайт и выбрать раздел, связанный с операциями над векторами. Затем в предложенную форму вести условие задания и запустить расчёт нажатием одной кнопки.

    Из множества онлайн-расчётчиков, доступных в секторе рунета, можно выделить следующие:

    • SolverBook — это простой на вид сайт, содержащий на своей странице приложение, позволяющее выполнять любые действия над отрезками, а также определять их вид. Кроме непосредственного предоставления ответа, сервис выдаёт пошаговое решение. При этом каждый этап будет детально расписан.
    • O nlineMSchool — сайт помогает найти коллинеарные отрезки для любой сложности примеров. На страницах сервиса находится вся необходимая теория и примеры решения заданий. Поэтому даже слабо подготовленный пользователь сможет разобраться во всех нюансах решения нужных ему задач.
    • Kontrolnaya-rabota — отличительной его чертой является возможность отправления подробного решения на указанную электронную почту. j ;

      2) |k |=1, но | i x j | = |i | |J | sin(90°)=1;

      3) векторы i , j и k образуют правую тройку (см. рис. 16).

      7.2. Свойства векторного произведения

      1. При перестановке сомножителей векторное произведение меняет знак, т.е. а хb =(b хa ) (см. рис. 19).

      Векторы а хb и b ха коллинеарны, имеют одинаковые модули (площадь параллелограмма остается неизменной), но противоположно направлены (тройки а , b , а хb и a , b , b x a противоположной ориентации). Стало быть a xb = -(b xa ).

      2. Векторное произведение обладает сочетательным свойством относительно скалярного множителя, т. е. l (а хb ) = (l а ) х b = а х (l b ).

      Пусть l >0. Вектор l (а хb ) перпендикулярен векторам а и b . Вектор ( l а )хb также перпендикулярен векторам а и b (векторы а , l а лежат в одной плоскости). Значит, векторы l (а хb ) и ( l а )хb коллинеарны. Очевидно, что и направления их совпадают. Имеют одинаковую длину:

      Поэтому l (a хb )= l а хb . Аналогично доказывается при l

      3. Два ненулевых вектора а и b коллинеарны тогда и только тогда, когда их векторное произведение равно нулевому вектору, т. е. а ||b а хb =0 .

      В частности, i *i =j *j =k *k =0 .

      4. Векторное произведение обладает распределительным свойством:

      (a +b ) хс = а хс +b хс .

      Примем без доказательства.

      7.3. Выражение векторного произведения через координаты

      Мы будем использовать таблицу векторного произведения векторов i , j и k :

      если направление кратчайшего пути от первого вектора к второму совпадает с направлением стрелки, то произведение равно третьему вектору, если не совпадает — третий вектор берется со знаком «минус».

      Пусть заданы два вектора а =а х i +a y j +a z k и b =b x i +b y j +b z k . Найдем векторное произведение этих векторов, перемножая их как многочлены (согласно свойств векторного произведения):



      Полученную формулу можно записать еще короче:

      так как правая часть равенства (7.1) соответствует разложению определителя третьего порядка по элементам первой строки.Равенство (7.2) легко запоминается.

      7.4. Некоторые приложения векторного произведения

      Установление коллинеарности векторов

      Нахождение площади параллелограмма и треугольника

      Согласно определению векторного произведения векторов а и b |а хb | = |а | * |b |sin g , т. е. S пар = |а х b |. И, значит, D S =1/2|а х b |.

      Определение момента силы относительно точки

      Пусть в точке А приложена сила F =АВ и пусть О — некоторая точка пространства (см. рис. 20).

      Из физики известно, что моментом си лы F относительно точки О называется вектор М , который проходит через точку О и:

      1) перпендикулярен плоскости, проходящей через точки О, А, В;

      2) численно равен произведению силы на плечо

      3) образует правую тройку с векторами ОА и A В .

      Стало быть, М =ОА х F .

      Нахождение линейной скорости вращения

      Скорость v точки М твердого тела, вращающегося с угловой скоростью w вокруг неподвижной оси, определяется формулой Эйлера v =w хr , где r =ОМ , где О-некоторая неподвижная точка оси (см. рис. 21).

      Угол между векторами

      Для того чтобы мы могли ввести понятие векторного произведения двух векторов, нужно сначала разобраться с таким понятие, как угол между этими векторами.

      Пусть нам даны два вектора $\overline{α}$ и $\overline{β}$. Возьмем в пространстве какую-либо точку $O$ и отложим от нее векторы $\overline{α}=\overline{OA}$ и $\overline{β}=\overline{OB}$, тогда угол $AOB$ будет называться углом между этими векторами (рис. 1).

      Обозначение: $∠(\overline{α},\overline{β})$

      Понятие векторного произведения векторов и формула нахождения

      Определение 1

      Векторным произведением двух векторов называется вектор, перпендикулярный обоим данным векторам, и его длина будет равняться произведению длин этих векторов с синусом угла между данными векторами, а также этот вектор с двумя начальными имеют туже ориентацию, как и декартова система координат.\circ=4\cdot 3\cdot 1=12$

      Ответ: $12$.

      Вычисление векторного произведения по координатам векторов

      Из определения 1 сразу же вытекает и способ нахождения векторного произведения для двух векторов. Поскольку вектор кроме значения имеет еще и направление, находить его только при помощи скалярной величины невозможно. Но помимо него существует еще способ нахождения с помощью координат данных нам векторов.

      Пусть нам даны векторы $\overline{α}$ и $\overline{β}$, которые будут иметь координаты $(α_1,α_2,α_3)$ и $(β_1,β_2,β_3)$, соответственно. Тогда вектор векторного произведения (а именно его координаты) можно найти по следующей формуле:

      $\overline{α}х\overline{β}=\begin{vmatrix}\overline{i}&\overline{j}&\overline{k}\\α_1&α_2&α_3\\β_1&β_2&β_3\end{vmatrix}$

      Иначе, раскрывая определитель, получим следующие координаты

      $\overline{α}х\overline{β}=(α_2 β_3-α_3 β_2,α_3 β_1-α_1 β_3,α_1 β_2-α_2 β_1)$

      Пример 2

      Найти вектор векторного произведения коллинеарных векторов $\overline{α}$ и $\overline{β}$ с координатами $(0,3,3)$ и $(-1,2,6)$.

      Решение .

      Воспользуемся формулой, приведенной выше. Получим

      $\overline{α}х\overline{β}=\begin{vmatrix}\overline{i}&\overline{j}&\overline{k}\\0&3&3\\-1&2&6\end{vmatrix}=(18-6)\overline{i}-(0+3)\overline{j}+(0+3)\overline{k}=12\overline{i}-3\overline{j}+3\overline{k}=(12,-3,3)$

      Ответ: $(12,-3,3)$.

      Свойства векторного произведения векторов

      Для произвольных смешанных трех векторов $\overline{α}$, $\overline{β}$ и $\overline{γ}$, а также $r∈R$ справедливы следующие свойства:

      Пример 3

      Найдите площадь параллелограмма, вершины которого имеют координаты $(3,0,0)$, $(0,0,0)$, $(0,8,0)$ и $(3,8,0)$.

      Решение .

      Вначале изобразим данный параллелограмм в координатном пространстве (рис.5):

      Рисунок 5. Параллелограмм в координатном пространстве. Автор24 — интернет-биржа студенческих работ

      Видим, что две стороны этого параллелограмма построены с помощью коллинеарных векторов с координатами $\overline{α}=(3,0,0)$ и $\overline{β}=(0,8,0)$.2}=24$

      Использование векторного произведения ВЕКТОРОВ

      для вычисления площади

      некоторых геометрических фигур

      Исследовательская работа по математике

      Ученика 10 Б класса

      МОУ СОШ №73

      Перевозникова Михаила

      Руководители:

      Ассистент каф. математического анализа механико-математического факультета СГУ им. Н.Г. Чернышевского Бердников Глеб Сергеевич

      Саратов, 2015

      Введение.

      1. Теоретический обзор.

      1.1. Векторы и вычисления с векторами.

      1.2. Использование скалярного произведения векторов в решении задач

      1.3 Скалярное произведение векторов в координатах

      1.4. Векторное произведение векторов в трёхмерном Евклидовом пространстве: определение понятия.

      1.5. Координаты векторного произведения векторов.

      2. Практическая часть.

      2.1. Связь векторного произведения с площадью треугольника и параллелограмма. Выведение формулы и геометрический смысл векторного произведения векторов.

      2.2. Зная только координаты точек, найти площадь треугольника. Доказательство теоремы

      2.3. Проверка на примерах правильности формулы.

      2.4. Практическое использование векторной алгебры и произведения векторов.

      Заключение

      Введение

      Как известно, многие геометрические задачи имеют два ключевых способа решения – графический и аналитический. Графический метод связан с построением графиков и чертежей, а аналитический предполагает решение задач преимущественно с помощью алгебраических действий. В последнем случае алгоритм решений задач связан с аналитической геометрией. Аналитическая геометрия – это область математики, а точнее линейной алгебры, которая рассматривает решение геометрических задач средствами алгебры на основе метода координат на плоскости и в пространстве. Аналитическая геометрия позволяет анализировать геометрические образы, исследовать линии и поверхности, важные для практических приложений. При этом в этой науке для расширения пространственного понимания фигур помимо иногда применяется векторное произведение векторов.

      В связи с широким распространением трехмерных пространственных технологий, изучение свойств некоторых геометрических фигур с использованием векторного произведения представляется актуальным.

      В связи с этим была обозначена цель данного проекта – использование векторного произведения векторов для вычисления площади некоторых геометрических фигур.

      В связи с поставленной целью решались следующие задачи:

      1. Теоретически изучить необходимые основы векторной алгебры и дать определение векторному произведению векторов в системе координат;

      2. Проанализировать наличие связи векторного произведения с площадью треугольника и параллелограмма;

      3. Вывести формулу площади треугольника и параллелограмма в координатах;

      4. Проверить на конкретных примерах верность выведенной формулы.

      1. Теоретический обзор.

        1. Векторы и вычисления с векторами

      Векторомназывается направленный отрезок, для которого указано его начало и конец:

      В данном случае началом отрезка является точка А , концом отрезка – точка В . Сам вектор обозначен через
      или . Чтобы найти координаты вектора
      , зная координаты его начальной точек А и конечной точки В, необходимо из координат конечной точки вычесть соответствующие координаты начальной точки:

      = { B x — A x ; B y — A y }

      Коллинеарными называются векторы, лежащие на параллельных прямых или на одной прямой. При этом вектор отрезок, характеризующийся длиной и направлением.

      Длина направленного отрезка определяет числовое значение вектора и называется длиной вектора или модулем вектора.

      Длина вектора || в прямоугольных декартовых координатах равна квадратному корню из суммы квадратов его координат.

      С векторами можно совершать различные действия.

      Например, сложение. Чтобы их сложить, нужно провести сначала второй вектор из конца первого, а потом соединить начало первого с концом второго (рис. 1). Суммой векторов является другой вектор с новыми координатами.

      Сумму векторов = {a x ; a y } и = {b x ; b y } можно найти воспользовавшись следующей формулой:

      + = {a x + b x ; a y + b y }

      Рис. 1. Действия с векторами

      Вычитая векторы, нужно сначала провести их из одной точки, а потом соединить конец второго с концом первого.

      Разность векторов = {a x ; a y } и = {b x ; b y } можно найти по формуле:

      — = { a x — b x ; a y — b y }

      Также, векторы можно умножать на число. Результатом также будет вектор, который в k раз больше (или меньше) данного. Его направление будет зависеть от знака k: при положительном k векторы сонаправлены, а при отрицательном – противоположно направлены.

      Произведение вектора = {a x ; a y } и числа k можно найти воспользовавшись следующей формулой:

      k · = {k · a x ; k · a y }

      А можно ли умножать вектор на вектор? Конечно, и даже двумя вариантами!

      Первый вариант – скалярное произведение.

      Рис. 2. Скалярное произведение в координатах

      Для нахождения произведения векторов можно использовать угол  между данными векторами, показанный на рисунке 3.

      Из формулы следует, что скалярное произведение равно произведению длин данных векторов на косинус угла между ними, его результатом является число. Важно, что если векторы перпендикулярны, то их скалярное произведение равно нулю, т.к. косинус прямого угла между ними равен нулю.

      В координатной плоскости вектор также имеет координаты. Вектора, их координаты и скалярное произведение являются одними из самых удобных методов вычисления угла между прямыми (или их отрезками), если введена система координат. И если координаты
      , то их скалярное произведение равно:

      В трехмерном пространстве существует 3 оси и, соответственно, у точек и векторов в такой системе будет по 3 координаты, а скалярное произведение векторов вычисляется по формуле:

      1.2. Векторное произведение векторов в трехмерном пространстве.

      Вторым вариантом вычисления произведения векторов является векторное произведение. Но, чтобы его определить требуется уже не плоскость, а трехмерное пространство, в котором начало и конец вектора имеют по 3 координаты.

      В отличие от скалярного произведения векторов в трёхмерном пространстве операция «векторное умножение» над векторами приводит к иному результату. Если в предыдущем случае скалярного умножения двух векторов результатом было число, то в случае векторного умножения векторов результатом будет другой вектор, перпендикулярный обоим вступившим в произведение векторам. Поэтому это произведение векторов называется векторным.

      Очевидно, что при построении результирующего вектора , перпендикулярного двум, вступившим в произведение — и , может быть выбрано два противоположных направления. При этом направление результирующего вектора определяется по правилу правой руки, или правилу буравчика.Если нарисовать векторы так, чтобы их начала совпадали и вращать первый вектор-сомножитель кратчайшим образом ко второму вектору-сомножителю, а четыре пальца правой руки показывали направление вращения (как бы охватывая вращающийся цилиндр), то оттопыренный большой палец покажет направление вектора-произведения (рис. 7).

      Рис. 7. Правило правой руки

      1.3. Свойства векторного произведения векторов.

      Длина результирующего вектора определяется по формуле

      .

      При этом
      векторное произведение. Как было сказано выше, результирующий вектор будет перпендикулярен
      , а его направление определяется по правилу правой руки.

      Векторное произведение зависит от порядка сомножителей, именно:

      Векторное произведение ненулевых векторов равно 0, если они коллинеарны, тогда синус угла между ними будет равен 0.

      Координаты векторов в трехмерном пространстве выражаются следующим образом: . Тогда координаты результирующего вектора находим по формуле

      Длина результирующего вектора находится по формуле:

      .

      2. Практическая часть.

      2.1. Связь векторного произведения с площадью треугольника и параллелограмма в плоскости. Геометрический смысл векторного произведения векторов.

      Пусть нам дан треугольник ABC (рис. 8). Известно, что .

      Если представить стороны треугольника АВ и АС в виде двух векторов, то в формуле площади треугольника мы находим выражение векторного произведения векторов:

      Из выше сказанного можно определить геометрический смысл векторного произведения (рис. 9):

      длина векторного произведения векторов равна удвоенной площади треугольника, имеющего сторонами векторы и , если их отложить от одной точки.

      Другими словами, длина векторного произведения векторов и равна площади параллелограмма, построенного на векторах и , со сторонами и и углом между ними, равным .


      Рис. 9. Геометрический смысл векторного произведения векторов

      В связи с этим, можно привести еще одно определение векторного произведения векторов:

      Векторным произведением вектора на вектор называется вектор , длина которого численно равна площади параллелограмма построенного на векторах и , перпендикулярный к плоскости этих векторов и направленный так, чтоб наименьшее вращение от к вокруг вектора осуществлялось против часовой стрелки, если смотреть с конца вектора (рис. 10).


      Рис. 10. Определение векторного произведения векторов

      с использованием параллелограмма

      2.2. Вывод формулы для нахождения площади треугольника в координатах.

      Итак, нам дан треугольник АВС в плоскости и координаты его вершин. Найдем площадь этого треугольника (рис. 11).

      Рис. 11. Пример решения задачи на нахождение площади треугольника по координатам его вершин

      Решение.

      Для начала, рассмотрим координаты вершин в пространстве и вычислим координаты векторов АВ и АС.

      По данной прежде формуле подсчитаем координаты их векторного произведения. Длина этого вектора равна 2 площадям треугольника АВС. Площадь треугольника равна 10.

      Более того, если мы рассмотрим треугольник на плоскости, то первые 2 координаты векторного произведения всегда будут равны нулю, поэтому мы можем сформулировать следующую теорему.

      Теорема: Пусть дан треугольник АВС и координаты его вершин (рис. 12).

      Тогда .

      Рис. 12. Доказательство теоремы

      Доказательство.

      Рассмотрим точки в пространстве и вычислим координаты векторов ВС и ВА. . По приведенной раньше формуле вычислим координаты векторного произведения этих векторов. Обратим внимание, что все члены, содержащие z 1 или z 2, равны 0, т.к. z 1и z 2 = 0. УБРАТЬ!!!

      Итак, следовательно,

      2.3. Проверка правильности формулы на примерах

      Найти площадь треугольника образованного векторами a = {-1; 2; -2} и b = {2; 1; -1}.

      Решение: Найдем векторное произведение этих векторов:

      a × b=

      I(2 · (-1) — (-2) · 1) — j((-1) · (-1) — (-2) · 2) + k((-1) · 1 — 2 · 2) =

      I(-2 + 2) — j(1 + 4) + k(-1 — 4) = -5 j — 5 k = {0; -5; -5}

      Из свойств векторного произведения:

      SΔ =

      | a × b| =

      √ 02 + 52 + 52 =

      √ 25 + 25 =

      √ 50 =

      5√ 2

      Ответ: SΔ = 2.5√2.

      Заключение

      2.4. Приложения векторной алгебры

      и скалярного и векторного произведения векторов.

      Где же нужны векторы? Векторное пространство и векторы носят не только теоретический характер, но и имеют вполне реальное практическое применение в современном мире.

      В механике и физике многие величины имеют не только численное значение, но и направление. Такие величины называются векторными. Вместе с использованием элементарных механических понятий, опираясь на их физический смысл, многие величины рассматриваются как скользящие векторы, а их свойства описываются как аксиомами, как это принято в теоретической механике, так и при помощи математических свойств векторов. Наиболее яркими примерами векторных величин являются скорость, импульс и сила (рис. 12). Например, момент импульса и сила Лоренца математически записываются с помощью векторов.

      В физике важны не только сами вектора, но в большой степени важны и их произведения, которые помогают вычислять некоторые величины. Векторное произведение полезно для определения коллинеарности векторов модуль векторного произведения двух векторов равен произведению их модулей, если они перпендикулярны, и уменьшается до нуля, если векторы сонаправленны или противоположно направленны.

      Еще один пример: скалярное произведение используется для вычисления работы по приведенной ниже формуле, где F – вектор силы, а s – вектор перемещения.

      Одним из примеров использования произведения векторов является момент силы, равный произведению радиус-вектора, проведенного от оси вращения к точке приложения силы, на вектор этой силы.

      Многое из того, что вычисляется в физике по правилу правой руки является векторным произведением. Найти подтверждения, привести примеры.

      Стоит еще заметить, что двухмерным и трехмерным пространством не исчерпываются возможные варианты векторных пространств. Высшая математика рассматривает пространства большей размерности, в которых также определяются аналоги формул для скалярного и векторного произведения. Несмотря на то, что пространства большей размерности, чем 3, человеческое сознание неспособно представить визуально, они удивительным образом находят себе приложения во многих областях науки и промышленности.

      В то же время результатом векторного произведения векторов в трёхмерном Евклидовом пространстве является не число, а результирующий вектор со своими координатами, направлением и длиной.

      Направление результирующего вектора определяется по правилу правой руки, что является одним из самых удивительных положений аналитической геометрии.

      Векторное произведение векторов может быть использовано в нахождении площади треугольника или параллелограмма по заданным координатам вершин, что было подтверждено выведением формулы, доказательством теоремы и решением практических задач.

      Векторы широко используются в физике, где такие показатели как скорость, импульс и сила могут быть представлены в виде векторных величин и вычисляются геометрически.

      Список использованных источников

      Атанасян Л. С., Бутузов В. Ф., Кадомцев С. Б. и др. Геометрия. 7-9 классы: учебник для общеобразовательных организаций. М.: , 2013. 383 с.

      Атанасян Л.С., Бутузов В. Ф., Кадомцев С. Б. и др. Геометрия. 10-11 классы: учебник для общеобразовательных организаций: базовый и профильный уровни. М.: , 2013. 255 с.

      Бугров Я.С., Никольский С.М. Высшая математика. Том первый: элементы линейной алгебры и аналитической геометрии.

      Клетеник Д.В. Сборник задач по аналитической геометрии. М.: Наука, Физматлит, 1998.

      Аналитическая геометрия.

      Математика. Клевер.

      Изучение математики онлайн.

      http://ru.onlinemschool.com/math/library/vector/multiply1/

      Сайт В. Глазнева.

      http://glaznev.sibcity.ru/1kurs/analit/common/html/anlek7.htm

      Википедия.

      https://ru.wikipedia.org/wiki/%C2%E5%EA%F2%EE%F0%ED%EE%E5_%EF%F0%EE%E8%E7%E2%E5%E4%E5%ED%E8%E5

      На данном уроке мы рассмотрим ещё две операции с векторами: векторное произведение векторов и смешанное произведение векторов (сразу ссылка, кому нужно именно оно) . Ничего страшного, так иногда бывает, что для полного счастья, помимо скалярного произведения векторов , требуется ещё и ещё. Такая вот векторная наркомания. Может сложиться впечатление, что мы залезаем в дебри аналитической геометрии. Это не так. В данном разделе высшей математики вообще мало дров, разве что на Буратино хватит. На самом деле материал очень распространенный и простой – вряд ли сложнее, чем то же скалярное произведение , даже типовых задач поменьше будет. Главное в аналитической геометрии, как многие убедятся или уже убедились, НЕ ОШИБАТЬСЯ В ВЫЧИСЛЕНИЯХ. Повторяйте как заклинание, и будет вам счастье =)

      Если векторы сверкают где-то далеко, как молнии на горизонте, не беда, начните с урока Векторы для чайников , чтобы восстановить или вновь приобрести базовые знания о векторах. Более подготовленные читатели могут знакомиться с информацией выборочно, я постарался собрать максимально полную коллекцию примеров, которые часто встречаются в практических работах

      Чем вас сразу порадовать? Когда я был маленьким, то умел жонглировать двумя и даже тремя шариками. Ловко получалось. Сейчас жонглировать не придётся вообще, поскольку мы будем рассматривать только пространственные векторы , а плоские векторы с двумя координатами останутся за бортом. Почему? Такими уж родились данные действия – векторное и смешанное произведение векторов определены и работают в трёхмерном пространстве. Уже проще!

      В данной операции, точно так же, как и в скалярном произведении, участвуют два вектора . Пусть это будут нетленные буквы .

      Само действие обозначается следующим образом: . Существуют и другие варианты, но я привык обозначать векторное произведение векторов именно так, в квадратных скобках с крестиком.

      И сразу вопрос : если в скалярном произведении векторов участвуют два вектора, и здесь тоже умножаются два вектора, тогда в чём разница ? Явная разница, прежде всего, в РЕЗУЛЬТАТЕ:

      Результатом скалярного произведения векторов является ЧИСЛО:

      Результатом векторного произведения векторов является ВЕКТОР : , то есть умножаем векторы и получаем снова вектор. Закрытый клуб. Собственно, отсюда и название операции. В различной учебной литературе обозначения тоже могут варьироваться, я буду использовать букву .

      Определение векторного произведения

      Сначала будет определение с картинкой, затем комментарии.

      Определение : Векторным произведением неколлинеарных векторов , взятых в данном порядке , называется ВЕКТОР , длина которого численно равна площади параллелограмма , построенного на данных векторах; вектор ортогонален векторам , и направлен так, что базис имеет правую ориентацию:

      Разбираем определение по косточкам, тут много интересного!

      Итак, можно выделить следующие существенные моменты:

      1) Исходные векторы , обозначенные красными стрелками, по определению не коллинеарны . Случай коллинеарных векторов будет уместно рассмотреть чуть позже.

      2) Векторы взяты в строго определённом порядке : – «а» умножается на «бэ» , а не «бэ» на «а». Результатом умножения векторов является ВЕКТОР , который обозначен синим цветом. Если векторы умножить в обратном порядке, то получим равный по длине и противоположный по направлению вектор (малиновый цвет). То есть, справедливо равенство .

      3) Теперь познакомимся с геометрическим смыслом векторного произведения. Это очень важный пункт! ДЛИНА синего вектора (а, значит, и малинового вектора ) численно равна ПЛОЩАДИ параллелограмма, построенного на векторах . На рисунке данный параллелограмм заштрихован чёрным цветом.

      Примечание : чертёж является схематическим, и, естественно, номинальная длина векторного произведения не равна площади параллелограмма.

      Вспоминаем одну из геометрических формул: площадь параллелограмма равна произведению смежных сторон на синус угла между ними . Поэтому, исходя из вышесказанного, справедлива формула вычисления ДЛИНЫ векторного произведения:

      Подчёркиваю, что в формуле речь идёт о ДЛИНЕ вектора, а не о самом векторе . Каков практический смысл? А смысл таков, что в задачах аналитической геометрии площадь параллелограмма часто находят через понятие векторного произведения:

      Получим вторую важную формулу. Диагональ параллелограмма (красный пунктир) делит его на два равных треугольника. Следовательно, площадь треугольника, построенного на векторах (красная штриховка), можно найти по формуле:

      4) Не менее важный факт состоит в том, что вектор ортогонален векторам , то есть . Разумеется, противоположно направленный вектор (малиновая стрелка) тоже ортогонален исходным векторам .

      5) Вектор направлен так, что базис имеет правую ориентацию. На уроке о переходе к новому базису я достаточно подробно рассказал об ориентации плоскости , и сейчас мы разберёмся, что такое ориентация пространства. Объяснять буду на пальцах вашей правой руки . Мысленно совместите указательный палец с вектором и средний палец с вектором . Безымянный палец и мизинец прижмите к ладони. В результате большой палец – векторное произведение будет смотреть вверх. Это и есть правоориентированный базис (на рисунке именно он). Теперь поменяйте векторы (указательный и средний пальцы ) местами, в результате большой палец развернётся, и векторное произведение уже будет смотреть вниз. Это тоже правоориентированный базис. Возможно, у вас возник вопрос: а какой базис имеет левую ориентацию? «Присвойте» тем же пальцам левой руки векторы , и полУчите левый базис и левую ориентацию пространства (в этом случае большой палец расположится по направлению нижнего вектора) . Образно говоря, данные базисы «закручивают» или ориентируют пространство в разные стороны. И это понятие не следует считать чем-то надуманным или абстрактным – так, например, ориентацию пространства меняет самое обычное зеркало, и если «вытащить отражённый объект из зазеркалья», то его в общем случае не удастся совместить с «оригиналом». Кстати, поднесите к зеркалу три пальца и проанализируйте отражение;-)

      …как всё-таки хорошо, что вы теперь знаете о право- и левоориентированных базисах, ибо страшнЫ высказывания некоторых лекторов о смене ориентации =)

      Векторное произведение коллинеарных векторов

      Определение подробно разобрано, осталось выяснить, что происходит, когда векторы коллинеарны. Если векторы коллинеарны, то их можно расположить на одной прямой и наш параллелограмм тоже «складывается» в одну прямую. Площадь такого, как говорят математики, вырожденного параллелограмма равна нулю. Это же следует и из формулы – синус нуля или 180-ти градусов равен нулю, а значит, и площадь нулевая

      Таким образом, если , то и . Обратите внимание, что само векторное произведение равно нулевому вектору, но на практике этим часто пренебрегают и пишут, что оно тоже равно нулю.

      Частный случай – векторное произведение вектора на самого себя:

      С помощью векторного произведения можно проверять коллинеарность трёхмерных векторов, и данную задачу среди прочих мы тоже разберём.

      Для решения практических примеров может потребоваться тригонометрическая таблица , чтобы находить по ней значения синусов.

      Ну что же, разжигаем огонь:

      Пример 1

      а) Найти длину векторного произведения векторов , если

      б) Найти площадь параллелограмма, построенного на векторах , если

      Решение : Нет, это не опечатка, исходные данные в пунктах условия я намеренно сделал одинаковыми. Потому что оформление решений будет отличаться!

      а) По условию требуется найти длину вектора (векторного произведения). По соответствующей формуле:

      Ответ :

      Коль скоро спрашивалось о длине, то в ответе указываем размерность – единицы.

      б) По условию требуется найти площадь параллелограмма, построенного на векторах . Площадь данного параллелограмма численно равна длине векторного произведения:

      Ответ :

      Обратите внимание, что в ответе о векторном произведении речи не идёт вообще, нас спрашивали о площади фигуры , соответственно, размерность – квадратные единицы.

      Всегда смотрим, ЧТО требуется найти по условию, и, исходя из этого, формулируем чёткий ответ. Может показаться буквоедством, но буквоедов среди преподавателей хватает, и задание с хорошими шансами вернётся на доработку. Хотя это не особо натянутая придирка – если ответ некорректен, то складывается впечатление, что человек не разбирается в простых вещах и/или не вник в суть задания. Этот момент всегда нужно держать на контроле, решая любую задачу по высшей математике, да и по другим предметам тоже.

      Куда подевалась большая буковка «эн»? В принципе, её можно было дополнительно прилепить в решение, но в целях сократить запись, я этого не сделал. Надеюсь, всем понятно, что и – это обозначение одного и того же.

      Популярный пример для самостоятельного решения:

      Пример 2

      Найти площадь треугольника, построенного на векторах , если

      Формула нахождения площади треугольника через векторное произведение дана в комментариях к определению. Решение и ответ в конце урока.

      На практике задача действительно очень распространена, треугольниками вообще могут замучить.

      Для решения других задач нам понадобятся:

      Свойства векторного произведения векторов

      Некоторые свойства векторного произведения мы уже рассмотрели, тем не менее, я их включу в данный список.

      Для произвольных векторов и произвольного числа справедливы следующие свойства:

      1) В других источниках информации данный пункт обычно не выделяют в свойствах, но он очень важен в практическом плане. Поэтому пусть будет.

      2) – свойство тоже разобрано выше, иногда его называют антикоммутативностью . Иными словами, порядок векторов имеет значение.

      3) – сочетательные или ассоциативные законы векторного произведения. Константы безпроблемно выносятся за пределы векторного произведения. Действительно, чего им там делать?

      4) – распределительные или дистрибутивные законы векторного произведения. С раскрытием скобок тоже нет проблем.

      В качестве демонстрации рассмотрим коротенький пример:

      Пример 3

      Найти , если

      Решение: По условию снова требуется найти длину векторного произведения. Распишем нашу миниатюру:

      (1) Согласно ассоциативным законам, выносим константы за переделы векторного произведения.

      (2) Выносим константу за пределы модуля, при этом модуль «съедает» знак «минус». Длина же не может быть отрицательной.

      (3) Дальнейшее понятно.

      Ответ :

      Пора подбросить дров в огонь:

      Пример 4

      Вычислить площадь треугольника, построенного на векторах , если

      Решение : Площадь треугольника найдём по формуле . Загвоздка состоит в том, что векторы «цэ» и «дэ» сами представлены в виде сумм векторов. Алгоритм здесь стандартен и чем-то напоминает примеры № 3 и 4 урока Скалярное произведение векторов . Решение для ясности разобьём на три этапа:

      1) На первом шаге выразим векторное произведение через векторное произведение , по сути, выразим вектор через вектор . О длинах пока ни слова!

      (1) Подставляем выражения векторов .

      (2) Используя дистрибутивные законы, раскрываем скобки по правилу умножения многочленов.

      (3) Используя ассоциативные законы, выносим все константы за пределы векторных произведений. При маломальском опыте действия 2 и 3 можно выполнять одновременно.

      (4) Первое и последнее слагаемое равно нулю (нулевому вектору) благодаря приятному свойству . Во втором слагаемом используем свойство антикоммутативности векторного произведения:

      (5) Приводим подобные слагаемые.

      В результате вектор оказался выражен через вектор, чего и требовалось достичь:

      2) На втором шаге найдем длину нужного нам векторного произведения. Данное действие напоминает Пример 3:

      3) Найдём площадь искомого треугольника:

      Этапы 2-3 решения можно было оформить и одной строкой.

      Ответ :

      Рассмотренная задача достаточно распространена в контрольных работах, вот пример для самостоятельного решения:

      Пример 5

      Найти , если

      Краткое решение и ответ в конце урока. Посмотрим, насколько вы были внимательны при изучении предыдущих примеров;-)

      Векторное произведение векторов в координатах , заданных в ортонормированном базисе , выражается формулой :

      Формула и правда простецкая: в верхнюю строку определителя записываем координатные векторы, во вторую и третью строки «укладываем» координаты векторов , причём укладываем в строгом порядке – сначала координаты вектора «вэ», затем координаты вектора «дубль-вэ». Если векторы нужно умножить в другом порядке, то и строки следует поменять местами:

      Пример 10

      Проверить, будут ли коллинеарны следующие векторы пространства:
      а)
      б)

      Решение : Проверка основана на одном из утверждений данного урока: если векторы коллинеарны, то их векторное произведение равно нулю (нулевому вектору): .

      а) Найдём векторное произведение:

      Таким образом, векторы не коллинеарны.

      б) Найдём векторное произведение:

      Ответ : а) не коллинеарны, б)

      Вот, пожалуй, и все основные сведения о векторном произведении векторов.

      Данный раздел будет не очень большим, так как задач, где используется смешанное произведение векторов, немного. Фактически всё будет упираться в определение, геометрический смысл и пару рабочих формул.

      Смешанное произведение векторов – это произведение трёх векторов :

      Вот так вот они выстроились паровозиком и ждут, не дождутся, когда их вычислят.

      Сначала опять определение и картинка:

      Определение : Смешанным произведением некомпланарных векторов , взятых в данном порядке , называется объём параллелепипеда , построенного на данных векторах, снабжённый знаком «+», если базис правый, и знаком «–», если базис левый.

      Выполним рисунок. Невидимые нам линии прочерчены пунктиром:

      Погружаемся в определение:

      2) Векторы взяты в определённом порядке , то есть перестановка векторов в произведении , как вы догадываетесь, не проходит без последствий.

      3) Перед тем, как прокомментировать геометрический смысл, отмечу очевидный факт: смешанное произведение векторов является ЧИСЛОМ : . В учебной литературе оформление может быть несколько другим, я привык обозначать смешанное произведение через , а результат вычислений буквой «пэ».

      По определению смешанное произведение – это объем параллелепипеда , построенного на векторах (фигура прочерчена красными векторами и линиями чёрного цвета). То есть, число равно объему данного параллелепипеда.

      Примечание : чертёж является схематическим.

      4) Не будем заново париться с понятием ориентации базиса и пространства. Смысл заключительной части состоит в том, что к объёму может добавляться знак минус. Простыми словами, смешанное произведение может быть отрицательным: .

      Непосредственно из определения следует формула вычисления объема параллелепипеда, построенного на векторах .

      Векторное произведение — это псевдовектор, перпендикулярный плоскости, построенной по двум сомножителям, являющийся результатом бинарной операции «векторное умножение» над векторами в трёхмерном Евклидовом пространстве. Векторное произведение не обладает свойствами коммутативности и ассоциативности (является антикоммутативным) и, в отличие от скалярного произведения векторов, является вектором. Широко используется во многих технических и физических приложениях. Например, момент импульса и сила Лоренца математически записываются в виде векторного произведения. Векторное произведение полезно для «измерения» перпендикулярности векторов — модуль векторного произведения двух векторов равен произведению их модулей, если они перпендикулярны, и уменьшается до нуля, если векторы параллельны либо антипараллельны.

      Определить векторное произведение можно по-разному, и теоретически, в пространстве любой размерности n можно вычислить произведение n-1 векторов, получив при этом единственный вектор, перпендикулярный к ним всем. Но если произведение ограничить нетривиальными бинарными произведениями с векторным результатами, то традиционное векторное произведение определено только в трёхмерном и семимерном пространствах. Результат векторного произведения, как и скалярного, зависит от метрики Евклидова пространства.

      В отличие от формулы для вычисления по координатам векторов скалярного произведения в трёхмерной прямоугольной системе координат, формула для векторного произведения зависит от ориентации прямоугольной системы координат или, иначе, её «хиральности».

      Определение:
      Векторным произведением вектора a на вектор b в пространстве R 3 называется вектор c , удовлетворяющий следующим требованиям:
      длина вектора c равна произведению длин векторов a и b на синус угла φ между ними:
      |c|=|a||b|sin φ;
      вектор c ортогонален каждому из векторов a и b;
      вектор c направлен так, что тройка векторов abc является правой;
      в случае пространства R7 требуется ассоциативность тройки векторов a,b,c.
      Обозначение:
      c===a × b


      Рис. 1. Площадь параллелограмма равна модулю векторного произведения

      Геометрические свойства векторного произведения :
      Необходимым и достаточным условием коллинеарности двух ненулевых векторов является равенство нулю их векторного произведения.

      Модуль векторного произведения равняется площади S параллелограмма, построенного на приведённых к общему началу векторах a и b (см. рис.1).

      Если e — единичный вектор, ортогональный векторам a и b и выбранный так, что тройка a,b,e — правая, а S — площадь параллелограмма, построенного на них (приведённых к общему началу), то для векторного произведения справедлива формула:
      =S e


      Рис.2. Объём параллелепипеда при использовании векторного и скалярного произведения векторов; пунктирные линии показывают проекции вектора c на a × b и вектора a на b × c, первым шагом является нахождение скалярных произведений

      Если c — какой-нибудь вектор, π — любая плоскость, содержащая этот вектор, e — единичный вектор, лежащий в плоскости π и ортогональный к c,g — единичный вектор, ортогональный к плоскости π и направленный так, что тройка векторов ecg является правой, то для любого лежащего в плоскости π вектора a справедлива формула:
      =Pr e a |c|g
      где Pr e a проекция вектора e на a
      |c|-модуль вектора с

      При использовании векторного и скалярного произведений можно высчитать объём параллелепипеда, построенного на приведённых к общему началу векторах a, b и c . Такое произведение трех векторов называется смешанным.
      V=|a (b×c)|
      На рисунке показано, что этот объём может быть найден двумя способами: геометрический результат сохраняется даже при замене «скалярного» и «векторного» произведений местами:
      V=a×b c=a b×c

      Величина векторного произведения зависит от синуса угла между изначальными векторами, поэтому векторное произведение может восприниматься как степень «перпендикулярности» векторов также, как и скалярное произведение может рассматриваться как степень «параллельности». Векторное произведение двух единичных векторов равно 1 (единичному вектору), если изначальные векторы перпендикулярны, и равно 0 (нулевому вектору), если векторы параллельны либо антипараллельны.

      Выражение для векторного произведения в декартовых координатах
      Если два вектора a и b определены своими прямоугольными декартовыми координатами, а говоря точнее — представлены в ортонормированном базисе
      a=(a x ,a y ,a z)
      b=(b x ,b y ,b z)
      а система координат правая, то их векторное произведение имеет вид
      =(a y b z -a z b y ,a z b x -a x b z ,a x b y -a y b x)
      Для запоминания этой формулы:
      i =∑ε ijk a j b k
      где ε ijk — символ Леви-Чивиты.

      Как определить, являются ли точки коллинеарными в координатной геометрии?

      Как доказать, что точки коллинеарны в координатной геометрии?

      Определение коллинеарных точек:

      • Три или более точки, лежащие на одной прямой, называются коллинеарными точками.
      • Рассмотрим прямую линию L в указанной выше декартовой координатной плоскости, образованную осью x и осью y.
      • Эта прямая L проходит через три точки A, B и C, координаты которых равны (2, 4), (4, 6) и (6, 8) соответственно.
      • {Альтернативно мы можем сказать, что три точки A (2, 4), B (4, 6) и C (6, 8) лежат на одной прямой L}
      • Три или более точек, которые лежащие на одной прямой, называются коллинеарными точками.

      Как определить, коллинеарны ли три точки ?:

      • Есть два метода определения коллинеарности трех точек.
      • Один — это метод формулы наклона, а другой — метод площади треугольника.
      • Метод формулы наклона для определения коллинеарности точек.
      • Три или более точки коллинеарны, если наклон любых двух пар точек одинаков.
      • С тремя точками A, B и C могут быть образованы три пары точек: AB, BC и AC.
      • Если наклон AB = наклон BC = наклон AC, то A, B и C являются коллинеарными точками.

      Пример

      Покажите, что три точки A (2, 4), B (4, 6) и C (6, 8) лежат на одной прямой.

      Решение:

      • Если три точки A (2, 4), B (4, 6) и C (6, 8) коллинеарны, то
      • наклонов любых двух пар точек будут равны.
      • Теперь примените формулу наклона, чтобы найти наклоны соответствующих пар точек:
      • Наклон AB = (6–4) / (4–2) = 1,
      • Наклон BC = (8–6) / (6-4) = 1, и
      • Наклон AC = (8-4) / (6-2) = 1
      • Поскольку наклоны любых двух пар из трех пар точек одинаковы, это доказывает, что A, B и C — коллинеарные точки.
      • Площадь треугольника для определения коллинеарности трех точек.
      • Три точки коллинеарны, если значение площади треугольника, образованного тремя точками, равно нулю.
      • Примените координаты указанных трех точек в формуле площади треугольника. Если результат для площади равен нулю, то данные точки называются коллинеарными.
      • Прежде всего, вспомним формулу площади треугольника, образованного тремя точками.

      Это

      В приведенной выше формуле две вертикальные полосы, охватывающие переменные, представляют определитель.

      Давайте применим координаты трех вышеуказанных точек A, B и C в приведенной выше формуле определителя для площади треугольника, чтобы проверить, равен ли ответ нулю.

      Так как результат для площади треугольника равен нулю, то точки A (2, 4), B (4, 6) и C (6, 8) коллинеарны.

      Коллинеарные 3-х мерные линии

      Коллинеарные точки — 2D

      Все коллинеарные точки расположены на одной прямой.

      Точки будут коллинеарны, если:

      И после перестановки членов во избежание деления на ноль получаем:

      (x 2 — x 1 ) (y 3 — y 1 ) — (x 3 — x 1 ) (y 2 — y 1 ) = 0

      Другой способ проверить, являются ли точки коллинеарными, — это вычислить площадь, образованную точками, если площадь равна нулю, то точки коллинеарны.

      И после перестановки членов и исключения значения 1/2 получаем условие коллинеарности:

      x 1 (y 2 — y 3 ) + x 2 (y 3 — y 1 ) + x 3 (y 1 — y 2 ) = 0

      Коллинеарные точки — 3D

      Точка 1: (x 1 , y 1 , z 1 )
      Точка 2: (x 2 , y 2 , z 2 )
      Точка 3: (x 3 , y 3 , z 3 )

      Пример: Найдите, лежат ли следующие точки на одной прямой.

      (-1, 0, 2), (1, 1, 4), (3, 2, 6)

      Согласно векторному кросс-произведению:

      (1-0) (6-2) — (2-0) (4-2) = 0
      (3 + 1) (4-2) — (1 + 1) (6-2) ) = 0
      (1 + 1) (2-0) — (3 + 1) (1-0) = 0

      Все значения равны 0, поэтому точки коллинеарны (лежат на одной линии).

      Проверка сторон треугольника:

      а = 3 б = 6 с = 3

      Видно, что неравенство треугольника не выполняется

      А также площадь A = 0.

      , следовательно, точки лежат на одной прямой.

      Коллинеарные точки — точки, расположенные на одной прямой.

      Можно применить несколько методов, чтобы определить, являются ли точки коллинеарными

      1. Если векторное произведение векторов n 1 и n 2 равно нулю во всех направлениях тогда точки коллинеарны, n 1 и n 2 — векторы, соединяющие одну точку с два других пункта. Это похоже на вычисление площади треугольника путем перекрестного произведения.
      Первый вектор: n 1 = (x 2 — x 1 ) i + (y 2 — y 1 ) j + (z 2 — z 1 ) k
      Второй вектор: n 2 = (x 3 — x 1 ) i + (y 3 — y 1 ) j + (z 3 — z 1 ) к

      И произведение в направлениях x, y и z:

      (y 2 — y 1 ) (z 3 — z 1 ) — (y 3 — y 1 ) (z 2 — z 1 ) = 0
      (x 3 — x 1 ) (z 2 — z 1 ) — (x 2 — x 1 ) (z 3 — z 1 ) = 0
      (x 2 — x 1 ) (y 3 — y 1 ) — (x 3 — x 1 ) (y 2 — y 1 ) = 0

      Условие колинеарности: n 1 × n 2 = 0 (0i + 0j + 0k) .

      2. Вычисление сторон треугольника a, b и c по уравнениям:

      Теперь примените один из следующих методов:

      неравенство треугольника: a + b> c a + c> b b + c> a

      , если все неравенства верны, то точки не лежат на одной прямой.

      Второй метод — вычисление площади по формуле Герона:

      Если площадь равна 0, то точки лежат на одной прямой (s — половина периметра).

      Численные ограничения достижения ориентации геологических плоскостей

      В статье обсуждаются ограничения аналитического определения положения геологической плоскости с помощью трех неколлинеарных точек.Мы представляем проблемы, которые возникают при вычислении ориентации плоскости, порожденной почти коллинеарными точками. Мы отнесли эти ошибки к неточностям арифметики с плавающей запятой. Чтобы продемонстрировать проблему, мы исследовали поверхность с постоянной ориентацией. Мы использовали триангуляцию Делоне для расчета параметров ее локальной ориентации. Мы ввели новую меру коллинеарности, применимую для определения положения плоских треугольников. Используя эту меру, мы показали, что определенные плоскости, созданные триангуляцией, нельзя рассматривать как надежный источник измерения.Чтобы изучить взаимосвязь между коллинеарностью и ориентацией, мы использовали комбинаторный алгоритм для получения всех возможных плоскостей из заданного набора точек. Предложен статистический критерий отбраковки почти коллинеарных плоскостей.

      1 Введение

      Определение ориентации геологических плоскостей с использованием трех неколлинеарных точек широко известно как «трехточечная задача». Есть несколько подходов, которые касаются вычислительных аспектов этой концепции [1,2,3,4].Естественное применение «трехточечной задачи» может быть связано с графической оценкой ориентации геологических горизонтов при геологическом картировании [5, 6]. Он также используется для определения ориентации трещин с помощью тахеометра без отражателя [7], а также для сбора и анализа основных стратиграфических и структурных данных с помощью («кибер-картографии») [8]. Ориентацию исследуемых поверхностей можно также проверить с помощью ортофотопланов, лидаров или цифровой фотограмметрии [9,10,11,12,13,14,15].Существуют подходы, которые сосредотачиваются на извлечении плоскостей из неструктурированных трехмерных облаков точек с использованием методов быстрого перемещения или kd-деревьев [16]. Многие из вышеперечисленных методов используют концепцию наиболее подходящей плоскости, основанную на подходе наименьших квадратов [7, 9, 10, 14,15,16] или анализе момента инерции [17,18,19,20]. Методы, связанные с приложениями получения наиболее подходящей плоскости, все еще развиваются с точки зрения поиска лучших критериев для выбора наиболее подходящей плоскости [21]. Как правило, подход наименьших квадратов кажется адекватным при рассмотрении плоскости, положение которой почти одинаково распределено.Однако ожидается, что анализ момента инерции определит максимальную плотность векторов [20], которая больше относится к частоте столкновения с определенным положением, а не к расстоянию от средней ориентации в рамках рассматриваемого набора измерений. В этом исследовании мы сосредотачиваемся на ограничениях получения локального отношения в рамках рассматриваемого набора трехмерных точек. Для получения локального отношения мы использовали триангуляцию Делоне. Такой подход требует оценки входных данных, потому что вычисление конкретных параметров «плоского» треугольника (вершины которого почти коллинеарны) может быть неточным из-за неточностей арифметики с плавающей запятой.Пример такой ошибки представлен Голдбергом [22], который показал, что вычисление площади длинного и тонкого («плоского») треугольника с использованием формулы Герона (которая использует только длины треугольника) дает относительно большие неточности. Поэтому они предложили переписать эту формулу, чтобы получить более точные результаты. Другой пример арифметических ошибок округления с плавающей запятой касается вычисления центра тяжести треугольника, который не обязательно является плоским. Вопреки математической теории, может появиться много результатов в соответствии с разной последовательностью вершин, i.e ., abc , bca или cab . Эти два примера могут называться числовой ненадежностью . Ошибки арифметического округления с плавающей запятой также могут приводить к геометрическому вырождению, которое связано с ошибочным вычислением геометрических структур (, например, , триангуляция Делоне, диаграммы Вороного). Такая ошибка представляет собой проблемы геометрической ненадежности [23].

      Количественные ограничения на коллинеарность при вычислении положения геологических плоскостей были предоставлены Фернандесом [20].Этот подход можно рассматривать как модификацию анализа моментов инерции, однако некоторые исследователи указывают, что он может не иметь числовой устойчивости [24]. Основное отличие состоит в том, что Фернандес использовал трехмерные данные с географической привязкой, а не направляющие косинусы векторов нормали к плоскостям, как предполагалось изначально. Они ввели центр масс точек (среднее значение координат X Y Z) и построили векторы, связав этот центр масс с каждой точкой. Затем рассчитывалась «матрица ориентации» с использованием направляющих косинусов этих векторов.Степень коллинеарности обозначена K и рассчитана следующим образом:

      K знак равно л п ( λ 1 / λ 2 ) / ( λ 2 / λ 3 ) (1)

      , где λ 1 , λ 2 , λ 3 — собственные значения «матрицы ориентации».Они ввели порог коллинеарности K <0,8, который, однако, в некоторых случаях считался слишком ограничивающим [24]. Более того, этот подход не относил проблему коллинеарности к ошибкам округления. Он был скорее основан на предположении, что для набора коллинеарных точек многие плоскости наилучшего соответствия могут рассматриваться с одинаковой степенью соответствия.

      Цель этого исследования — представить более явную и интуитивно понятную меру коллинеарности, которую можно отнести к конкретным плоскостям, а не к пространственному распределению точек.Мы предполагаем, что эта мера должна охватывать весь интервал изменчивости коллинеарности. В этом исследовании мы рассматриваем надежность измерений, полученных с помощью алгоритма триангуляции Делоне. Однако наиболее известным свойством этого алгоритма является то, что он максимизирует минимальный угол среди всех сгенерированных треугольников [25]. Таким образом, вероятность возникновения коллинеарных конфигураций при вычислении треугольников Делоне ниже. Таким образом, мы использовали комбинаторный алгоритм, чтобы получить больше треугольников, подверженных коллинеарности [26].Он генерирует все трехэлементные наборы, которые можно выбрать из n-элементного набора. Другими словами, этот алгоритм генерирует все возможные плоскости, которые могут быть сгенерированы в рамках рассматриваемого набора точек. Мы отнесли к исследуемым плоскостям три параметра (коллинеарность, размер и угол падения) и показали взаимосвязь между ними. Мы предложили естественный критерий для определения диапазона допустимости введенного коэффициента коллинеарности по отношению к исходным данным.

      2 метода

      Чтобы исследовать ориентацию поверхности с почти одинаково распределенной ориентацией, мы использовали полевой контроллер Leica CS 15 GPS.Мы собрали координаты XYZ 90 произвольно выбранных точек, принадлежащих одному из покрытых травой берегов в центре Катовице, Польша, в непосредственной близости от Международного конгресс-центра (ICC). Хотя большинство этих берегов имеют пологий наклон (5-15 градусов), чтобы получить более понятную визуализацию взаимосвязи между выбранными параметрами, мы выбрали берег, угол падения которого составлял приблизительно 30 градусов. Использовалась модель эллипсоида GRS 1980 и ETRS89 (EPSG 2180) в качестве системы координат.Мы использовали хорошо зарекомендовавшее себя кинематическое позиционирование в реальном времени (RTK). Опорная станция имела следующие координаты: X = 266421,157, Y = 501902,300 и Z = 265,051. Измерения были произведены 22 ноября 2016 г., при сборе координат было видно не менее 14 спутников. Средняя точность получения координат составила 0,014 м для координат XY и 0,012 м для координаты Z.

      Чтобы правильно вычислить триангуляцию Делоне, мы использовали хорошо зарекомендовавшую себя библиотеку CGAL [27].Он включает алгоритмы вычислительной геометрии и поддерживает принцип Exact Geometric Computation (EGC). Принцип EGC обеспечивает надежные геометрические результаты, тем не менее, основным недостатком этого подхода является время [28]. Таким образом, мы использовали триангуляцию Делоне, доступную в библиотеке CGAL, и получили 170 плоскостей.

      Для вычисления их ориентации использовалась следующая процедура. Алгоритм требовал перекрестного произведения двух трехмерных векторов, представляющих плоскость.Следующую процедуру можно рассматривать как основу для вычисления угла падения плоскости, образованной тремя неколлинеарными точками.

      1. Предположим, у нас есть три точки:

        п 1 знак равно ( Икс 1 , у 1 , z 1 ) , (2а)

        п 2 знак равно ( Икс 2 , у 2 , z 2 ) , (2b)

        п 3 знак равно ( Икс 3 , у 3 , z 3 ) (2c)

      2. Строим два вектора, представляющих плоскость.Для упрощения координат мы используем буквы для координат в последующих расчетах.

        V 1 V → 2 знак равно [ Икс 2 — Икс 1 , у 2 — у 1 , z 2 — z 1 ] , ж ты р т час е р d е п о т е d б у [ а , б , c ] (3а)

        V 1 V → 3 знак равно [ Икс 3 — Икс 1 , у 3 — у 1 , z 3 — z 1 ] , ж ты р т час е р d е п о т е d б у [ d , е , ж ] (3b)

        V 2 V → 3 знак равно [ Икс 3 — Икс 2 , у 3 — у 2 , z 3 — z 2 ] , (3c)

      3. Мы вычисляем длины трех векторов и сортируем их в порядке возрастания.Таким образом, мы получаем вектор длин ( l 1 , l 2 , l 3 ) такой, что l 1 l 2 l 3 . Чтобы проверить, не лежат ли точки на одной прямой, воспользуемся неравенством треугольника. Он говорит, что в треугольнике длина самого длинного ребра должна быть меньше суммы двух оставшихся ребер. Кроме того, мы также можем получить коэффициент, который оценивает форму самолета.Этот коэффициент получается делением:

        d знак равно л 3 л 1 + л 2 (4)

        Очевидно, из приведенного выше определения сразу следует, что введенный коэффициент безразмерен.Мы видим, что коэффициент, вычисленный для равносторонних треугольников, равен 0,5, а для коллинеарных «треугольников» равен 1. Таким образом, коэффициент представляет собой число из интервала [0,5, 1] ​​и указывает степень равносторонности или коллинеарности. . Мы обозначаем коэффициент коллинеарности как d , но, эквивалентным образом, «уровень допуска», обозначенный как α = 1 — d , также можно рассматривать.

      4. Мы вычисляем кросс-вектор двух выбранных векторов V 1 ⃗V 2 , V 3 ⃗V 1 , используя определитель следующей матрицы:

        а d Икс б е Y c ж Z (5)

      5. Дано уравнение плоскости:

        ( б ж — c е ) Икс + ( c d — а ж ) у + ( а е — б d ) z знак равно 0 (6)

      6. Даны координаты вектора нормали N исследуемой плоскости:

        N знак равно [ б ж — c е , c d — а ж , а е — б d ] (7)

        Этот шаг требует проверки положительности третьей координаты.В противном случае все координаты необходимо умножить на –1. Это связано с тем, что вектор нормали к плоскости не определяется однозначно, и можно рассматривать два направления (рисунок 1). Но возьмем вектор, направленный вверх.

        Рисунок 1

        Самолет, образованный тремя точками. В расчетах плоскость представлена ​​нормальным (перпендикулярным) вектором. Угол падения рассматривается как угол между вектором нормали исследуемой плоскости и вектором, связанным с осью Z.Направление падения рассчитывается как угол между осью X, указывающей направление на север, и проекцией вектора нормали исследуемой плоскости на горизонтальную плоскость

      7. Пусть Z обозначает вектор, перпендикулярный исследуемой плоскости, скалярное произведение векторов обозначается символом «•» и ‖ K ‖ обозначает длину вектора K . Тогда угол падения рассчитывается следующим образом:

        а р c c о s | N ∙ Z | | | N | | | | Z | | (8)

        Направление падения требует для предлагаемого метода проекции вектора нормали исследуемой плоскости на горизонтальную плоскость.Таким образом, пусть H = ( h 1 , h 2 ) обозначает проекцию вектора N на горизонтальную плоскость. Чтобы вычислить азимут, мы использовали функцию atan2 , которая вычисляет угол между осью X и проецируемым вектором. Определение функции atan2 говорит, что для вектора t = [ u , v ] выражение atan 2 ( v , u ) измеряет угол между положительным Y -ось и точка ( u , v ).Таким образом, чтобы вычислить направление падения, мы должны вычислить atan 2 = ( h 1 , h 2 ). Формально функция atan2 работает следующим образом:

        а т а п 2 ( Икс , у ) знак равно а р c т а п ( Икс у ) я ж у > 0 , а р c т а п ( Икс у ) + π я ж у < 0 , Икс ≥ 0 , а р c т а п ( Икс у ) - π я ж у < 0 , + π 2 я ж у знак равно 0 , Икс > 0 , + π 2 я ж у знак равно 0 , Икс < 0 , ты п d е ж я п е d я ж Икс знак равно 0 , у знак равно 0.(9)

        Указанные выше параметры были отнесены к треугольникам Делоне (рис. 2). Тем не менее, распределение коэффициента коллинеарности предполагает, что количество коллинеарных плоскостей недостаточно для оценки интервала «взрыва» (Рисунок 3, Рисунок 4A).

        Рисунок 2

        Триангуляция Делоне исследуемых точек; Использовалась система координат ETRS89 (EPSG 2180)

        Рисунок 3

        Связь между коллинеарностью и углом падения треугольников Делоне

        Рисунок 4

        Распределение коллинеарности (A) Коллинеарность, приписываемая треугольникам Делоне (B) Коллинеарность, приписываемая 117 480 плоскостям, сгенерированным алгоритмом Липски

        Поскольку нашей целью было оценить максимально возможное значение коллинеарности, мы сгенерировали все трехэлементные подмножества из набора из девяноста элементов.Это было достигнуто с помощью алгоритма генерации подмножеств k -элементов из набора n -элементов [26]. По известному биномиальному коэффициенту мы получили 117 480 плоскостей. Поскольку график распределения показывает преобладание коллинеарных плоскостей (рис. 4В), мы можем более тщательно изучить взаимосвязь между коллинеарностью, размером (выраженным в м 2 ) и параметрами ориентации на следующих этапах.

      3 Результаты

      Мы расширили первоначальный набор измерений с помощью комбинаторного алгоритма и получили взаимосвязь между коллинеарностью, размером и углом падения (рис. 5A).Мы также включили некоторые частичные графики, которые описывают эту взаимосвязь в меньших интервалах. Угол падения имел относительно меньший разброс (около 12-15 градусов) среди равносторонних треугольников. Этот разброс можно значительно уменьшить (примерно до 1-3 градусов), если рассматривать только самые большие плоскости (рис. 5B). Визуальный эффект от этих результатов можно описать следующим образом: самые большие самолеты имели теплые (желто-красные) цвета и образовывали относительно узкий пояс. Этот пояс с обеих сторон окружали плоскости меньшего размера, которые можно было узнать по более холодным цветам.По ту сторону рассматриваемого интервала коллинеарности такого эффекта не происходило. При рассмотрении более коллинеарных конфигураций мы наблюдали большую случайность угла падения (рис. 5C). Однако даже для интервала [0,99750, 1] мы все еще могли распознать узкую полосу, представляющую ожидаемое значение угла падения. Потеря способности распознавать это значение может наблюдаться для плоскостей, коллинеарность которых больше, чем приблизительно 0,9999 (рис. 5D).

      Рисунок 5

      Связь между коллинеарностью, размером и углом падения для 117 480 плоскостей (A) Учитывается весь интервал коллинеарности (B) Рассматриваются равносторонние конфигурации (0.5 0,9975) (D) Представлено еще больше коллинеарных конфигураций (d> 0,9999)

      Таким образом, учет всех измерений повлиял бы на распределение, особенно на меры дисперсии (рис. 6А). Практические результаты этого исследования можно отнести к улучшению качества начального распределения угла наклона, генерируемого триангуляцией Делоне. Мы предложили наблюдать распределения угла наклона по возрастающей степени коллинеарности.Мы избегали установления новых произвольных коэффициентов универсального значения. Но поскольку исследуемая поверхность была относительно равномерно ориентирована, было несложно оценить интервал, в котором выбросы начали влиять на распределение (Рисунок 3, Рисунок 6A). Мы рассмотрели условное распределение и получили ожидаемые результаты: медиана в целом не изменилась; небольшое изменение должно наблюдаться для среднего значения, но наиболее значительное изменение произошло со стандартным отклонением (рис. 6A, рис. 6B).

      Рисунок 6

      (A) Распределение угла падения для всех треугольников Делоне (B) Условное распределение угла падения для треугольников Делоне, d которых меньше 0,975

      4 Обсуждение

      В этой статье мы исследовали влияние коллинеарности на надежность измерений ориентации геологических плоскостей. Предыдущие исследования, посвященные этой проблеме, относились к мере, основанной на собственных значениях «матрицы ориентации», и предлагались определенные ограничения.Этот подход, однако, больше относится к пространственному распределению точек, а не к конфигурации локальных плоскостей, положение которых исследуется. Более того, неточности арифметики с плавающей запятой не рассматривались как источник потенциальных ошибок. Основная цель этого исследования состояла в том, чтобы предоставить более интуитивную меру коллинеарности, которая охватывает весь интервал ее изменчивости. Это позволило изучить влияние этой особенности на числовую стабильность выходных данных. Однако при наложении числовых ограничений на входные данные, вероятно, всегда будет присутствовать впечатление произвольности.Тем не менее, для обнаружения выбросов мы предложили критерий, который больше относится к индивидуальному подходу, основанному на наблюдении за распределением измеряемого параметра. Вышеупомянутый подход может применяться до тех пор, пока поверхность ориентирована равномерно или ее ориентация известна a priori , так что выбросы могут быть легко обнаружены. В нашем исследовании мы исследовали ориентацию однородно ориентированной поверхности, которая была локально аппроксимирована плоскостями, порожденными триангуляцией Делоне. Основная цель этого подхода отличалась от того, что было рассмотрено в методе наилучшего соответствия.Ключевая проблема заключалась не в вычислении угла падения и направления падения, а в том, чтобы найти измерения, на которые влияют ошибки округления арифметических операций с плавающей запятой. Мы показали, что коллинеарность влияет на угол падения (, например, ., Рисунок 3, Рисунок 5A, Рисунок 6A), но из-за небольшого количества измерений мы не смогли описать взаимосвязь между выбранными параметрами. Чтобы исследовать это, мы использовали комбинаторный алгоритм, который предоставил большее количество плоскостей, затронутых коллинеарностью.Рисунок 5B показывает, что наименьшее отклонение от ожидаемого значения может быть отнесено к наибольшим плоскостям. Более того, маленькие плоскости равносторонней формы не влияли на угол падения так сильно, как коллинеарные плоскости относительно большего размера (Рисунок 5B, Рисунок 5D). Неинтуитивный вывод из этого исследования состоит в том, что распределение коллинеарности в пределах рассматриваемого набора точек может быть далеко от симметричного (рис. 4B), хотя рассматривались треугольники различной формы. Мы не рассматриваем этот асимметричный результат как общее правило при исследовании всех возможных плоскостей в пределах рассматриваемого набора данных.Тем не менее, этот пример показывает, что выбор произвольно выбранных плоскостей из набора данных и вычисление средней ориентации может привести к неожиданным результатам. Это связано с тем, что в нашем случае вероятность случайного выбора коллинеарной плоскости была относительно высокой, но мы указали, что коллинеарные конфигурации ненадежны с точки зрения получения положения. Таким образом, рассмотрение триангуляции Делоне кажется частичным решением этой проблемы. Тем не менее, чтобы уменьшить разброс измерений, полученных с помощью триангуляции Делоне, мы предложили ограничение.В нашем случае мы выбрали самое большое подмножество, которое не включает никаких выбросов. Таким образом, мы сократили интервал приемлемой коллинеарности (рис. 6В). В нашем случае мы устанавливаем порог коллинеарности равным 0,975, но мы не предполагаем, что это значение следует использовать во всех обстоятельствах. Поскольку большой разброс данных часто рассматривается как недостаток рассматриваемого подхода, наши результаты должны повысить надежность измерений ориентации в аналогичных геологических и методологических условиях.

      Я благодарен профессору Przemysław Koprowski, PhD Paweł Gładki за полезные обсуждения относительно оценки исходных данных, профессору Lesław Teper за полезные советы относительно последующих приложений и Michał Nowosiński за техническую помощь. Публикация частично финансируется из средств Ведущего национального исследовательского центра (KNOW), полученных Центром полярных исследований Силезского университета, Польша. Я также благодарен двум анонимным рецензентам, чьи проницательные комментарии значительно улучшили эту статью.

      Ссылки

      [1] Файнен М. Н. Задача трех точек, векторный анализ и расширение до задачи N точек. Journal of Geoscience Education, 2005, 53, 257–63. Поиск в Google Scholar

      [2] Де Паор Д.Г., Современное решение классической задачи трех точек. Журнал геологического образования, 1991, 1, 322–24. Поиск в Google Scholar

      [3] Вашер Х. Л., Задача трех точек в контексте элементарного векторного анализа. Журнал геологического образования, 1989, 8, 280–87. Поиск в Google Scholar

      [4] Ханеберг В.C., Метод лагранжевой интерполяции для трехточечных задач. Journal of Structural Geology, 1990, 12, 945-947, https://doi.org/10.1016/0191-8141(90)

    -B Поиск в Google Scholar

    [5] Рэган Д.М., Структурная геология: Введение в Геометрические приемы. Издательство Кембриджского университета, 2009 г. Поиск в Google Scholar

    [6] Грошонг Р. Х., Трехмерная структурная геология. Springer-Verlag Berlin Heidelberg, 2006 г. Поиск в Google Scholar

    [7] Feng Q., Sjögren P., Stephansson O., Цзин Л., Измерение ориентации трещин на открытых скалах с помощью тахеометра без отражателя. Англ. Geol., 2001, 59, 133-146, https://dol.org/10.1016/S0013-7952(00)00070-3 Поиск в Google Scholar

    [8] Xu X., Bhattacharya JB, Davies RK, Aiken CL, Цифровое геологическое картирование песчаника Феррон, Мадди-Крик, штат Юта, с помощью GPS и безотражательных лазерных дальномеров. Решения GPS, 2001, 5, 15-23, https://doi.org/10.1007/PL00012872 Поиск в Google Scholar

    [9] Cracknell M.Дж., Роуч М., Грин Д., Люсьер А., Оценка ориентации напластования на основе цифровых моделей рельефа с высоким разрешением. IEEE Transactions on Geoscience and Remote Sensing, 2013, 51, 2949-2959, https://doi.org/10.1109/TGRS.2012.2217502 Поиск в Google Scholar

    [10] Банерджи С., Митра С., Картографирование удаленной поверхности с использованием ортофотопланы и геологические карты, наложенные на цифровые модели рельефа: приложение к антиклинали Овец-Маунтин, Вайоминг. Бюллетень AAPG, 2004, 88, 1227-1237 Искать в Google Scholar

    [11] Martinez-Torres L.М., Лопетеги А., Эгуилуз Л. Автоматическое решение трехточечной геологической задачи. Компьютеры и науки о Земле, 2012, 42, 200-202, https://doi.org/10.1016/j.cageo.2011.08.031 Поиск в Google Scholar

    [12] Hasbargen LE, Тест трехточечного векторного метода для определения простирания и падения с использованием цифровых аэрофотоснимков и топографии. Специальные статьи Геологического общества Америки, 2012 г., 492, 199-208, http://dx.doi.org/10.1130/2012.2492(14) Поиск в Google Scholar

    [13] McCaffrey K.Дж. У., Джонс Р. Р., Холдсворт Р. Э., Уилсон Р. У., Клегг П., Имбер Дж., Холлиман Н., Тринкс И., Открытие пространственного измерения: цифровые технологии и будущее полевых исследований в области геолого-геофизических исследований. Журнал геологического общества, 2005, 162, 927-938, https://doi.org/10.1144/0016-764905-017 Поиск в Google Scholar

    [14] Бистакки А., Гриффит В.А., Смит С.А., Ди Торо Г., Джонс Р., Нильсен С., Шероховатость разлома на сейсмогенных глубинах по данным LIDAR и фотограмметрического анализа. Чистая и прикладная геофизика, 2011, 168, 2345-2363, https: // doi.org / 10.1007 / s00024-011-0301-7 Поиск в Google Scholar

    [15] Ассали П., Груссенмейер П., Виллемин Т., Поллет Н., Вигье Ф., Сплошные изображения для геоструктурного картирования и моделирования ключевых блоков неоднородности горных пород. Компьютеры и науки о Земле, 2016, 89, 21-31, https://doi.org/10.1016/j.cageo.2016.01.002 Поиск в Google Scholar

    [16] Девез Т.Дж., Жирардо-Монтау Д., Алланик К. , Ромер Дж., Facets: плагин CloudCompare для извлечения геологических плоскостей из неструктурированных трехмерных облаков точек.Международный архив фотограмметрии, дистанционного зондирования и пространственной информации, 2016 г., стр. 41, 799-804, https://doi.org/10.5194/isprs-archives-XLI-B5-799-2016 Поиск в Google Scholar

    [17 ] Шайдеггер А.Е., О статистике ориентации плоскостей напластования, осей зерен и аналогичных седиментологических данных. Газета профессионала геологической службы США, 1965, 525, 164–167. Поиск в Google Scholar

    [18] Уотсон Г. С. Экваториальные распределения на сфере. Биометрика, 1965, 52, 193-201, https: // doi.org / 10.1093 / biomet / 52.1-2.193 Поиск в Google Scholar

    [19] Вудкок Н. Х., Определение форм ткани с использованием метода собственных значений. Бюллетень Геологического общества Америки, 1977, 88, 1231-1236, https://doi.org/10.1130/0016-7606(1977)88%3C1231:SOFSUA%3E2.0.CO;2 Поиск в Google Scholar

    [ 20] Фернандес О., Получение наиболее подходящей плоскости с помощью трехмерных данных с географической привязкой. Journal of Structural Geology, 2005, 27, 855–58, https://doi.org/10.1016/j.jsg.2004.12.004 Поиск в Google Scholar

    [21] Jones R.Р., Пирс М. А., Жакемин К., Уотсон Ф. Э., Надежные и наиболее подходящие плоскости на основе геопространственных данных. Geosphere, 2016, 12, 196-202, https://doi.org/10.1130/GES01247.1 Поиск в Google Scholar

    [22] Голдберг Д., Что каждый компьютерный ученый должен знать об арифметике с плавающей запятой. ACM Computing Surveys (CSUR), 1991, 23, 5-48, https://doi.org/10.1145/103162.103163 Поиск в Google Scholar

    [23] Мэй Г., Типпер Дж. К., Сюй Н., Числовая надежность в геометрической вычисление: пояснительное резюме.Прикладная математика и информационные науки, 2014, 8, 2717, http://dx.doi.org/10.12785/amis/080607 Поиск в Google Scholar

    [24] Провидец Т.Д., Ходжеттс Д., Вероятностные ограничения на наилучшее соответствие структурных линеаментов. плоская точность, полученная посредством численного анализа. Journal of Structural Geology, 2016, 82, 37-47, https://doi.org/10.1016/j.jsg.2015.11.004 Поиск в Google Scholar

    [25] de Berg M., Cheong O., van Kreveld М., Овермарс М., Вычислительная геометрия. Springer-Verlag Berlin Heidelberg, Berlin, Heidelberg, 2008 Поиск в Google Scholar

    [26] Lipski W., Комбинаторика для программистов. Wydawnictwa Naukowo-Techniczne, Варшава, 2004 г. Поиск в Google Scholar

    [27] Херт С., Сил М., Д. Д. Выпуклые оболочки и триангуляции Делоне. В: CGAL User and Reference Manual. Редакционная коллегия CGAL, выпуск 4.13, 2018 г. Поиск в Google Scholar

    [28] Фогель Э., Гальперин Д., Вейн Р., Компоновки CGAL и их приложения: Пошаговое руководство. Springer Science & Business Media, 2012 г., http://dx.doi.org/10.1007 / 978-3-642-17283-0 Искать в Google Scholar

    Поступила: -08-03 2018

    Принято: 2018-07-03

    Опубликовано в сети: 07.09.2018

    © 2018 M. Michalak, опубликовано De Gruyter.

    Эта работа находится под лицензией Creative Commons Attribution-NonCommercial-NoDerivatives 4.0 License.

    % PDF-1.4 % 1 0 объект > эндобдж 4 0 obj (Титульная страница) эндобдж 5 0 obj > эндобдж 8 0 объект (Предварительные мероприятия) эндобдж 9 0 объект > эндобдж 12 0 объект (Преимущества барицентрических координат) эндобдж 13 0 объект > эндобдж 16 0 объект (Обозначения и условные обозначения) эндобдж 17 0 объект > эндобдж 20 0 объект (Как использовать эту статью) эндобдж 21 0 объект > эндобдж 24 0 объект (Основы) эндобдж 25 0 объект > эндобдж 28 0 объект (Координаты) эндобдж 29 0 объект > эндобдж 32 0 объект (Линии) эндобдж 33 0 объект > эндобдж 36 0 объект (Уравнение линии) эндобдж 37 0 объект > эндобдж 40 0 объект (Сева и Менелай) эндобдж 41 0 объект > эндобдж 44 0 объект (Особые точки в барицентрических координатах) эндобдж 45 0 объект > эндобдж 48 0 объект (Стандартные стратегии) эндобдж 49 0 объект > эндобдж 52 0 объект (EFFT: перпендикулярные линии) эндобдж 53 0 объект > эндобдж 56 0 объект (Формула расстояния) эндобдж 57 0 объект > эндобдж 60 0 объект (Круги) эндобдж 61 0 объект > эндобдж 64 0 объект (Уравнение круга) эндобдж 65 0 объект > эндобдж 68 0 объект (Более хитрая тактика) эндобдж 69 0 объект > эндобдж 72 0 объект (Области и линии) эндобдж 73 0 объект > эндобдж 76 0 объект (Ненормализованные координаты) эндобдж 77 0 объект > эндобдж 80 0 объект (O, H и сильный EFFT) эндобдж 81 0 объект > эндобдж 84 0 объект (Формула Конвея) эндобдж 85 0 объект > эндобдж 88 0 объект (Несколько заключительных лемм) эндобдж 89 0 объект > эндобдж 92 0 объект (Примеры проблем) эндобдж 93 0 объект > эндобдж 96 0 объект (USAMO 2001/2) эндобдж 97 0 объект > эндобдж 100 0 объект (Проблема и решение) эндобдж 101 0 объект > эндобдж 104 0 объект (Комментарий) эндобдж 105 0 объект > эндобдж 108 0 объект (USAMO 2008/2) эндобдж 109 0 объект > эндобдж 112 0 объект (Проблема и решение) эндобдж 113 0 объект > эндобдж 116 0 объект (Комментарий) эндобдж 117 0 объект > эндобдж 120 0 объект (ISL 2001 G1) эндобдж 121 0 объект > эндобдж 124 0 объект (Проблема и решение) эндобдж 125 0 объект > эндобдж 128 0 объект (Комментарий) эндобдж 129 0 объект > эндобдж 132 0 объект (2012 WOOT PO4 / 7) эндобдж 133 0 объект > эндобдж 136 0 объект (Проблема и решение) эндобдж 137 0 объект > эндобдж 140 0 объект (Комментарий) эндобдж 141 0 объект > эндобдж 144 0 объект (ISL 2005 G5) эндобдж 145 0 объект > эндобдж 148 0 объект (Проблема и решение) эндобдж 149 0 объект > эндобдж 152 0 объект (Комментарий) эндобдж 153 0 объект > эндобдж 156 0 объект (USA TSTST 2011/4) эндобдж 157 0 объект > эндобдж 160 0 объект (Проблема и решение) эндобдж 161 0 объект > эндобдж 164 0 объект (Комментарий) эндобдж 165 0 объект > эндобдж 168 0 объект (USA TSTST 2011/2) эндобдж 169 0 объект > эндобдж 172 0 объект (Проблема и решение) эндобдж 173 0 объект > эндобдж 176 0 объект (Комментарий) эндобдж 177 0 объект > эндобдж 180 0 объект (Проблемы) эндобдж 181 0 объект > эндобдж 184 0 объект (Проблемы) эндобдж 185 0 объект > эндобдж 188 0 объект (Подсказки) эндобдж 189 0 объект > эндобдж 192 0 объект (Дополнительные проблемы из СС 2012) эндобдж 193 0 объект > эндобдж 196 0 объект (Учебник по векторам) эндобдж 197 0 объект > эндобдж 200 0 объект (Определения) эндобдж 201 0 объект > эндобдж 204 0 объект (Треугольник ABC) эндобдж 205 0 объект > эндобдж 208 0 объект (Особые очки) эндобдж 209 0 объект > эндобдж 212 0 объект (Таблица формул) эндобдж 213 0 объект > эндобдж 216 0 объект (Стандартные формулы) эндобдж 217 0 объект > эндобдж 220 0 объект (Больше непонятных формул) эндобдж 221 0 объект > эндобдж 224 0 объект (Другие особые моменты) эндобдж 225 0 объект > эндобдж 228 0 объект (Особые линии и круги) эндобдж 229 0 объект > эндобдж 232 0 объект (Библиография) эндобдж 233 0 объект > эндобдж 238 0 объектов> поток x ڍ VKo6`oP1DJbO; M

    Коллинеарные векторы и примеры — примечания Читать

    они — коллинеарные векторы. Это один из трех существующих типов векторов. Речь идет о векторах, которые находятся в одном направлении или в одном направлении. Это означает следующее: два или более вектора будут линейными, если расположены по прямой линии, параллельной друг другу.

    Вектор определяется как величина, применяемая к объекту, и характеризуется как направление, вкус и масштаб. Векторы могут быть найдены на плоскости или в пространстве и могут быть разных типов: коллинеарные векторы, параллельные векторы и параллельные векторы.

    Индекс

    • 1 холин вектор
    • 2 Характеристики
      • 2.1 Пример 1
      • 2.2 Пример 2
      • 2.3 Пример 1
    • 3-х многоуровневая векторная система
      • 3.1 Коллинеарные векторы с противоположными смыслами
      • 3.2 Коллинеарные векторы с одинаковым значением
      • 3.3 Векторные коллекторы одинаковой величины и противоположных значений
    • 4 Разница между коллинеарными и параллельными векторами
    • 5 Ссылки

    Коллинеарный вектор

    Вектор является линейным, если линия действия одного вектора точно такая же, как линия действия всех других векторов, независимо от размера и вкуса каждого вектора.

    Векторы используются как представления в различных областях, таких как математика, физика, алгебра, а также в геометрии, где векторы коллинеарны только тогда, когда направления совпадают, независимо от того, что они означают.

    Элемент

    — Два или более вектора являются линейными, если отношения между координатами одинаковы.

    Пример 1

    У нас есть вектор m = m_x; m_y и n = n_x; н_й. Коллинеарно, если:

    Пример 2

    — Два или более вектора являются линейными, если произведение или произведение вектора равно нулю (0).Это потому, что в системе координат каждый вектор характеризуется своими координатами, и если они пропорциональны друг другу, векторы будут линейными. Это выражается следующим образом:

    Пример 1

    У нас есть векторы a = (10, 5) и b = (6, 3). Чтобы определить, являются ли они коллинеарными, применяется определяющая теория, которая устанавливает равенство перекрестных произведений. Таким образом, вам необходимо:

    Коллинеарная векторная система

    Коллинеарные векторы представлены графически с использованием направления и вкуса этого — с учетом того, что они должны проходить через точку приложения и модуль, которые имеют определенный масштаб или длину.

    Колинеарная векторная система образуется, когда два или более вектора воздействуют на объект или объекты, представляют собой силу и действуют в одном направлении.

    Например, если к объектам приложены две коллинеарные силы, результат этого будет зависеть только от направления, в котором они действуют. Всего три корпуса, а именно:

    Коллинеарный вектор с противоположными смыслами

    Результат двух коллинеарных векторов равен этому числу:

    R = Σ F = F 1 + F 2.

    Пример

    Если две силы действуют на поезд F 1 = 40 Н и F 2 = 20 Н в противоположном направлении (как показано на рисунке), результат будет:

    R = Σ F = (- 40 Н) + 20 Н.

    R = — 20 Н.

    Коллинеарные векторы с одинаковым значением

    Величина результирующей силы будет равна сумме коллинеарных векторов:

    R = Σ F = F 1 + F 2.

    Пример

    Если две силы действуют на поезд F 1 = 35 Н и F 2 = 55 Н в одном направлении (как показано на рисунке), результат будет:

    R = Σ F = 35 N + 55N.

    R = 90 Н.

    Положительные результаты показывают, что коллинеарный вектор работает влево.

    Спинальный вектор той же величины и противоположных чувств

    Результат двух коллинеарных векторов будет таким же, как сумма коллинеарных векторов:

    R = Σ F = F 1 + F 2.

    Поскольку сила имеет ту же величину, но в противоположном направлении, то есть одна будет положительной, а другая отрицательной, при сложении двух сил результат будет равен нулю.

    Пример

    Если на поезд действуют две силы F 1 = -7 Н и F 2 = 7 Н, которые имеют одинаковую величину, но в противоположном направлении (как показано на рисунке), результат будет:

    R = Σ F = (-7 Н) + 7 Н.

    R = 0.

    Поскольку результат равен 0, это означает, что векторы уравновешены друг с другом и, следовательно, тело находится в равновесии или в состоянии покоя (оно не будет двигаться).

    Разница между коллинеарными и параллельными векторами

    Коллинеарные векторы характеризуются тем, что они имеют одинаковое направление на одной и той же прямой или потому, что они параллельны прямой; то есть они являются прямыми параллельными линейными векторами.

    С другой стороны, параллельные векторы определены, потому что они находятся в разных линиях действий, которые перехватываются в одной точке.

    Другими словами, они имеют одинаковое происхождение или прибытие — независимо от их модуля, направления или направления — и образуют между ними угол.

    Параллельные векторные системы решаются математическими или графическими методами, такими как метод силового параллелограмма и метод многоугольной силы. Благодаря этому будет определено результирующее значение вектора, которое показывает направление, в котором будет двигаться тело.

    По сути, основное различие между коллинеарными векторами и параллельными векторами — это линия действия, в которой они действуют: коллинеарные векторы действуют в одной и той же строке, одновременно являясь параллельными в разных.

    То есть коллинеарные векторы действуют в одной плоскости «X» или «Y»; и одновременные действия на обоих планах, начиная с одной и той же точки.

    Коллинеарные векторы не находятся в одной точке, а находятся в одно и то же время, потому что они параллельны друг другу.

    На левом изображении вы видите блок. Связаны веревкой и узлом, делящим ее на две части; когда их тянут в разных направлениях и с разной силой, блоки будут двигаться в одном направлении.

    Представлены два вектора, которые совпадают в точке (блоке), независимо от модуля, понимания или направления.

    И наоборот, на правом рисунке показан шкив, поднимающий коробку.Веревка представляет собой линию действия; при вытягивании на него действуют две силы (вектора): одна сила натяжения (при подъеме на блок) и другая сила, действующая на вес балки. Оба имеют одинаковое направление, но в противоположном направлении; не согласны ни в чем.

    Номер ссылки

    Как найти направление при сложении векторов? — Mvorganizing.org

    Как найти направление при сложении векторов?

    Направление результирующей можно определить, найдя угол, который она составляет с вектором север-юг или восток-запад.На диаграмме справа показан угол тета (Θ), отмеченный внутри треугольника сложения векторов. Этот угол тета — это угол, который получается в результате с западом.

    Как сложить два вектора, используя их компоненты?

    Опять же, компонентный метод сложения можно резюмировать следующим образом:

    1. Используя тригонометрию, найдите компоненту x и компоненту y для каждого вектора.
    2. Сложите оба x-компонента (по одному от каждого вектора), чтобы получить x-компонент от суммы.
    3. Сложите оба y-компонента (по одному от каждого вектора), чтобы получить y-компонент от суммы.

    Какое направление вектора?

    Направление вектора — это угол, который он образует с горизонтальной линией. Для определения направления вектора можно использовать одну из следующих формул: tanθ = yx, где x — горизонтальное изменение, а y — вертикальное изменение.

    Есть ли направление у единичных векторов?

    Векторные величины имеют направление и величину.Однако иногда интересует только направление вектора, а не его величина. В таких случаях для удобства векторы часто «нормализуются», чтобы иметь единичную длину.

    Как определить, параллельны ли два вектора?

    Два вектора A и B параллельны тогда и только тогда, когда они скалярно кратны друг другу. A = k B, k — постоянная, не равная нулю. Два вектора A и B перпендикулярны тогда и только тогда, когда их скалярное произведение равно нулю.

    Когда два вектора параллельны, их скалярное произведение равно?

    Максимальное значение для скалярного произведения возникает, когда два вектора параллельны друг другу (вся «сила» от обоих векторов направлена ​​в одном направлении), но когда два вектора перпендикулярны друг другу, значение скалярного произведения равен 0 (один вектор имеет нулевую силу, направленную в направлении другого.

    Что, если два вектора коллинеарны?

    Условие-2: — Два вектора коллинеарны, если отношение их координат одинаково. Это неверно, если один из компонентов равен нулю. Условие-3: — Два вектора коллинеарны, если их перекрестное произведение равно нулевому вектору.

    У параллельных векторов одинаковое направление?

    Два вектора параллельны, если они имеют одинаковое направление или находятся в совершенно противоположных направлениях. Теперь снова вспомним геометрическую интерпретацию скалярного умножения.Когда мы выполняли скалярное умножение, мы генерировали новые векторы, которые были параллельны исходным векторам (и друг другу в этом отношении).

    Как узнать, параллельны ли две параметрические линии?

    мы можем выбрать две точки на каждой линии (в зависимости от того, как представлены линии и уравнения), затем для каждой пары точек вычесть координаты, чтобы получить вектор смещения. Если два вектора смещения или направления кратны друг другу, линии параллельны.

    Как вы доказываете, что векторы коллинеарны?

    Доказать, что векторы a, b и c коллинеарны, если и только если векторы (a-b) и (a-c) параллельны. В противном случае, чтобы доказать коллинеарность векторов, мы должны доказать (a-b) = k (a-c), где k — постоянная.

    Все ли равные векторы коллинеарны?

    Равные векторы имеют одинаковые величины и направление. На приведенном выше рисунке векторы b и d являются равными векторами. Коллинеарные векторы — это два или более вектора, параллельных одной линии.Следовательно, векторы a и c коллинеарны, но не равны.

    В чем разница между коллинеарными и параллельными векторами?

    Параллельные векторы — это векторы, которые имеют одинаковую или параллельную опору. Они могут иметь равные или неодинаковые величины, а их направления могут быть одинаковыми или противоположными. Два вектора коллинеарны, если они имеют одинаковое направление, параллельны или антипараллельны.

    Коллинеарность и параллельность — это то же самое?

    В качестве прилагательных разница между коллинеарностью и параллельностью состоит в том, что коллинеарность лежит на одной прямой, в то время как параллельность одинаково удалена друг от друга во всех точках.

    Что такое коллинеарный пример?

    Три или более точки, лежащие на одной прямой, являются коллинеарными точками. Пример: точки A, B и C лежат на прямой m. Нет линии, проходящей через все три точки A, B и D.

    Могут ли две параллельные прямые быть на одной прямой?

    Некопланарные линии Две точки всегда лежат на одной прямой. Две пересекающиеся или параллельные прямые всегда лежат в одной плоскости. Если три точки лежат на одной прямой, то площадь треугольника, образованного этими точками, равна нулю.Два коллинеарных вектора либо имеют одинаковое направление, либо противоположные направления.

    Могут ли линии быть коллинеарными?

    . Линия, на которой лежат точки, особенно если она связана с геометрической фигурой, например, треугольником, иногда называется осью. Две точки тривиально лежат на одной прямой, поскольку две точки определяют прямую.

    Какова формула коллинеарных точек?

    Если A, B и C — три коллинеарные точки, то AB + BC = AC или AB = AC — BC или BC = AC — AB. Если площадь треугольника равна нулю, то точки называются коллинеарными точками.Если три точки (x1, y1), (x2, y2) и (x3, y3) коллинеарны, то [x1 (y2 — y3) + x2 (y3 — y1) + x3 (y1 — y2)] = 0.

    Как определить, лежат ли точки на одной прямой?

    Три или более точки коллинеарны, если наклон любых двух пар точек одинаков. С тремя точками A, B и C могут быть образованы три пары точек: AB, BC и AC. Если наклон AB = наклон BC = наклон AC, то точки A, B и C лежат на одной прямой.

    Всегда ли 3 точки на одной прямой?

    Коллинеарные точки — это точки, лежащие на линии.Любые две точки всегда коллинеарны, потому что их всегда можно соединить прямой линией. Три или более точек могут быть на одной прямой, но это не обязательно. Копланарные точки: группа точек, лежащих в одной плоскости, компланарна.

    Что такое определение неколлинеарных точек?

    : не коллинеарен: a: не лежат и не действуют в одной прямой линии неколлинеарных сил. б: отсутствие прямой в общих неколлинеарных плоскостях.

    Как узнать, лежат ли 4 точки на одной прямой?

    После сохранения посмотрите, существует ли такое же значение на карте и не отличается ли пара точек от той, которую вы обрабатываете в данный момент.Если они разные, то у вас есть набор из 4 точек, лежащих на одной прямой.

    Как доказать, что три точки являются коллинеарными векторами?

    Если ab + bc = ac, то три точки лежат на одной прямой. Сегменты линии могут быть преобразованы в векторы ab, bc и ac, где величина векторов равна длине соответствующих упомянутых сегментов линии.

    Как узнать, что три точки коллинеарны, используя расстояние?

    Как правило, три точки A, B и C коллинеарны, если сумма длин любых двух отрезков из AB, BC и CA равна длине оставшегося отрезка, то есть либо AB + BC = AC. либо AC + CB = AB, либо BA + AC = BC.

    % PDF-1.2 % 398 0 объект > эндобдж xref 398 298 0000000016 00000 н. 0000006312 00000 н. 0000016263 00000 п. 0000016481 00000 п. 0000016660 00000 п. 0000017117 00000 п. 0000017477 00000 п. 0000018006 00000 п. 0000018268 00000 п. 0000018458 00000 п. 0000018689 00000 п. 0000018952 00000 п. 0000019004 00000 п. 0000019217 00000 п. 0000019402 00000 п. 0000019597 00000 п. 0000019813 00000 п. 0000020174 00000 п. 0000020334 00000 п. 0000020510 00000 п. 0000020668 00000 н. 0000020871 00000 п. 0000021138 00000 п. 0000021342 00000 п. 0000021531 00000 п. 0000021731 00000 п. 0000021956 00000 п. 0000022176 00000 п. 0000022409 00000 п. 0000022629 00000 п. 0000022858 00000 п. 0000023048 00000 н. 0000023283 00000 п. 0000023497 00000 п. 0000023736 00000 п. 0000023906 00000 п. 0000024103 00000 п. 0000024324 00000 п. 0000024516 00000 п. 0000024696 00000 п. 0000024869 00000 п. 0000025076 00000 п. 0000025288 00000 п. 0000025522 00000 п. 0000025780 00000 п. 0000025951 00000 п. 0000026156 00000 п. 0000026360 00000 п. 0000026576 00000 п. 0000026780 00000 п. 0000027024 00000 п. 0000027194 00000 п. 0000027354 00000 п. 0000027563 00000 п. 0000027784 00000 п. 0000027958 00000 п. 0000028196 00000 п. 0000028391 00000 п. 0000028551 00000 п. 0000028785 00000 п. 0000028987 00000 п. 0000029153 00000 п. 0000029390 00000 н. 0000029566 00000 п. 0000029779 00000 п. 0000030014 00000 п. 0000030192 00000 п. 0000030395 00000 п. 0000030624 00000 п. 0000030808 00000 п. 0000030989 00000 п. 0000031154 00000 п. 0000031322 00000 п. 0000031546 00000 п. 0000031753 00000 п. 0000031991 00000 п. 0000032260 00000 п. 0000032469 00000 н. 0000032724 00000 п. 0000032913 00000 п. 0000033148 00000 п. 0000033441 00000 п. 0000033663 00000 п. 0000033835 00000 п. 0000034084 00000 п. 0000034283 00000 п. 0000034635 00000 п. 0000035150 00000 п. 0000035306 00000 п. 0000035358 00000 п. 0000035581 00000 п. 0000035861 00000 п. 0000036163 00000 п. 0000036337 00000 п. 0000036579 00000 п. 0000036631 00000 п. 0000036795 00000 п. 0000037041 00000 п. 0000037275 00000 п. 0000037480 00000 п. 0000037707 00000 п. 0000037914 00000 п. 0000038100 00000 п. 0000038328 00000 п. 0000038572 00000 п. 0000038773 00000 п. 0000039017 00000 п. 0000039220 00000 п. 0000039472 00000 п. 0000039710 00000 п. 0000039973 00000 п. 0000040233 00000 п. 0000040285 00000 п. 0000040492 00000 п. 0000040679 00000 п. 0000040942 00000 п. 0000041211 00000 п. 0000041418 00000 п. 0000041697 00000 п. 0000041942 00000 п. 0000042172 00000 п. 0000042439 00000 п. 0000042654 00000 п. 0000042823 00000 п. 0000043328 00000 п. 0000043521 00000 п. 0000043700 00000 п. 0000043868 00000 п. 0000044056 00000 п. 0000044265 00000 п. 0000044467 00000 п. 0000044679 00000 п. 0000044893 00000 п. 0000045085 00000 п. 0000045286 00000 п. 0000045553 00000 п. 0000045906 00000 п. 0000046093 00000 п. 0000046273 00000 п. 0000046471 00000 п. 0000046674 00000 п. 0000046853 00000 п. 0000047096 00000 п. 0000047270 00000 п. 0000047459 00000 п. 0000047668 00000 п. 0000047878 00000 н. 0000048071 00000 п. 0000048242 00000 п. 0000048430 00000 н. 0000048627 00000 н. 0000048792 00000 п. 0000049018 00000 п. 0000049194 00000 п. 0000049397 00000 п. 0000049615 00000 п. 0000050697 00000 п. 0000051279 00000 п. 0000051605 00000 п. 0000051828 00000 п. 0000052014 00000 п. 0000052211 00000 п. 0000052387 00000 п. 0000052613 00000 п. 0000052845 00000 п. 0000053042 00000 п. 0000053278 00000 п. 0000053507 00000 п. 0000053701 00000 п. 0000053923 00000 п. 0000054194 00000 п. 0000054422 00000 п. 0000054587 00000 п. 0000054760 00000 п. 0000054956 00000 п. 0000055144 00000 п. 0000055388 00000 п. 0000055627 00000 п. 0000055827 00000 п. 0000056034 00000 п. 0000056241 00000 п. 0000056419 00000 п. 0000056625 00000 п. 0000056831 00000 п. 0000057038 00000 п. 0000057322 00000 п. 0000057522 00000 п. 0000057767 00000 п. 0000058023 00000 п. 0000058045 00000 п. 0000058704 00000 п. 0000058726 00000 п. 0000059341 00000 п. 0000059561 00000 п. 0000059763 00000 п. 0000059992 00000 н. 0000060217 00000 п. 0000060407 00000 п. 0000060633 00000 п. 0000060854 00000 п. 0000061086 00000 п. 0000061294 00000 п. 0000061529 00000 п. 0000061762 00000 п. 0000061955 00000 п. 0000062178 00000 п. 0000062384 00000 п. 0000062590 00000 н. 0000062809 00000 п. 0000063039 00000 п. 0000063245 00000 п. 0000063453 00000 п. 0000064002 00000 п. 0000064356 00000 п. 0000065049 00000 п. 0000065268 00000 п. 0000065466 00000 п. 0000065686 00000 п. 0000065898 00000 п. 0000066102 00000 п. 0000066307 00000 п. 0000066506 00000 п. 0000066703 00000 п. 0000066914 00000 п. 0000067078 00000 п. 0000067252 00000 п. 0000067490 00000 н. 0000067661 00000 п. 0000067898 00000 п. 0000068138 00000 п. 0000068293 00000 п. 0000068585 00000 п. 0000068848 00000 п. 0000068900 00000 п. 0000069158 00000 п. 0000069321 00000 п. 0000069564 00000 п. 0000069759 00000 п. 0000069932 00000 н. 0000070197 00000 п. 0000070412 00000 п. 0000070668 00000 п. 0000070919 00000 п. 0000071157 00000 п. 0000071380 00000 п. 0000071594 00000 п. 0000071819 00000 п. 0000072048 00000 п. 0000072281 00000 п. 0000072528 00000 п. 0000072777 00000 п. 0000072970 00000 п. 0000073186 00000 п. 0000073208 00000 п. 0000073859 00000 п. 0000073881 00000 п. 0000074507 00000 п. 0000074529 00000 п. 0000075170 00000 п. 0000075373 00000 п. 0000075542 00000 п. 0000075713 00000 п. 0000075914 00000 п. 0000076234 00000 п. 0000076334 00000 п. 0000076806 00000 п. 0000076858 00000 п. 0000077162 00000 п. 0000077295 00000 п.

    Добавить комментарий

    Ваш адрес email не будет опубликован. Обязательные поля помечены *